Крок 1 - Медицина 2016 (буклет)

1 / 200
У хворого після перелому верхньої третини плечової кістки розвинувся параліч задньої групи м’язів плеча i передпліччя. Який нерв пошкоджено? The patient developed paralysis of the posterior group of muscles of the shoulder and forearm after a fracture of the upper third of the humerus. Which nerve was damaged?

Променевий Radial

Пахвовий Axillary

М’язово-шкірний Musculocutaneous

Серединний Middle

Ліктьовий Cubit

2 / 200
Хворому з ревматоїдним артритом тривалий час вводили гідрокортизон. У нього з’явилися гіперглікемія, поліурія, глюкозурія, спрага. Ці ускладнення лікування є наслідком активації процесу: A patient with rheumatoid arthritis was administered hydrocortisone for a long time. He developed hyperglycemia, polyuria, glucosuria, and thirst. These treatment complications are the result of process activation:

Глікогеноліз Glycogenolysis

Ліполіз Lipolysis

Глікогенез Glycogenesis

Глюконеогенез Gluconeogenesis

Гліколіз Glycolysis

3 / 200
Для корекції артеріального тиску при колаптоїдному стані хворому було введено мезатон. Який механізм дії даного препарату? Mezaton was administered to the patient to correct blood pressure in a colaptoid condition. What is the mechanism of action of this drug?

Стимулює β-адренорецептори Stimulates β-adrenoceptors

Стимулює α- і β-адренорецептори Stimulates α- and β-adrenoceptors

Блокує α-адренорецептори Blocks α-adrenoceptors

Блокує β-адренорецептори Blocks β-adrenoceptors

Стимулює α-адренорецептори Stimulates α-adrenoceptors

4 / 200
На електронній мікрофотографії представлені структури у вигляді відкритих міхурців, внутрішня поверхня яких вистелена одношаровим епітелієм, який утворений респіраторними та секреторними клітинами. Які це структури? The electron micrograph shows structures in the form of open vesicles, the inner surface of which is lined with a single-layered epithelium formed by respiratory and secretory cells. What are these structures?

Альвеолярні ходи Alveolar passages

Бронхіоли Bronchioles

Термінальні бронхіоли Terminal bronchioles

Альвеоли Alveoli

Ацинуси Acinus

5 / 200
В гістопрепараті представлений паренхіматозний орган, поверхневий шар кіркової речовини якого формують клубочки, утворені ендокриноцитами. Якому органу належить дана морфологічна ознака? The histopreparation shows a parenchymal organ, the surface layer of the cortical substance of which is formed by glomeruli formed by endocrinocytes. To which organ does this morphological feature belong?

Лімфатичний вузол Lymph node

Щитовидна залоза Thyroid

Яєчник Ovary

Наднирник Adrenal gland

Селезінка Spleen

6 / 200
Після переходу до змішаного харчування у новонародженої дитини виникла диспепсія з діареєю, метеоризмом, відставанням у розвитку. Біохімічна основа даної патології полягає у недостатності: After switching to a mixed diet, the newborn child developed dyspepsia with diarrhea, flatulence, and developmental delay. The biochemical basis of this pathology is a deficiency:

Лактази та целобіази Lactases and cellobiases

Ліпази та креатинкінази Lipases and creatine kinases

Сахарази та ізомальтази Sucrases and isomaltases

Трипсину та хімотрипсину Trypsin and chymotrypsin

Целюлази Cellulases

7 / 200
Хворому перед операцією було введено дитилін (лістенон) і проведено інтубацію. Дефіцит якого ферменту в організмі хворого подовжує дію м’язового релаксанту? Before the operation, the patient was administered ditilin (Listenone) and intubated. The deficiency of which enzyme in the patient's body prolongs the effect of the muscle relaxant?

Сукцинатдегідрогеназа Succinate dehydrogenase

Карбангідраза Carbonhydrase

K-Na-АТФ-аза K-Na-ATPase

Псевдохолінестераза Pseudocholinesterase

N-ацетилтрансфераза N-acetyltransferase

8 / 200
У хворого із захворюванням печінки виявлено зниження вмісту протромбіну в крові. Це призведе, перш за все, до порушення: In a patient with liver disease, a decrease in the content of prothrombin in the blood was detected. This will lead, first of all, to a violation:

Другої фази коагуляційного гемостазу Second phase of coagulation hemostasis

Судинно-тромбоцитарного гемостазу Vascular-platelet hemostasis

Антикоагулянтних властивостей крові Anticoagulant properties of blood

Фібринолізу Fibrinolysis

Першої фази коагуляційного гемостазу First phase of coagulation hemostasis

9 / 200
У хворого із сечокам’яною хворобою виникли нестерпні спастичні болі. Для попередження больового шоку йому ввели разом з атропіном наркотичний анальгетик, що не має спазмогенного ефекту. Який це був препарат? A patient with urolithiasis developed intolerable spastic pains. To prevent painful shock, he was administered a narcotic analgesic with atropine, which does not have a spasmogenic effect. What was it preparation?

Етилморфіну гідрохлорид Ethylmorphine hydrochloride

Морфіну гідрохлорид Morphine hydrochloride

Трамадол Tramadol

Промедол Promedol

Пірітрамід Piritramide

10 / 200
Яким буде скорочення м’язів верхньої кінцівки при намаганні підняти непосильний вантаж? What will be the contraction of the muscles of the upper limb when trying to lift an overwhelming load?

Ізометричние Isometric

Ізотонічне Isotonic

Фазичне Phase

Одиночне Single

Ауксотонічне Auxotonic

11 / 200
Для лікування урогенітальних інфекцій використовують хінолони - інгібітори ферменту ДнК-гірази. Який процес порушується під дією хінолонів у першу чергу? Quinolones are used to treat urogenital infections - inhibitors of the DnK-gyrase enzyme. What process is disrupted by quinolones in the first place?

Зворотна транскрипція Reverse transcription

Ампліфікація генів Gene Amplification

Реплікація ДНК DNA Replication

Рекомбінація генів Gene recombination

Репарація ДНК DNA Repair

12 / 200
Жінку госпіталізовано в клініку з симптомами гострого живота. При обстеженні виникла підозра на позаматкову вагітність. Яке з анатомічних утворень таза необхідно пропунктувати для підтвердження діагнозу? A woman was admitted to the clinic with symptoms of an acute abdomen. An ectopic pregnancy was suspected during the examination. Which of the pelvic anatomical formations must be identified to confirm the diagnosis?

Processus vaginalis peritonei Processus vaginalis peritonei

Fossa ischiorectalis Fossa ischiorectalis

Excavatio rectovesicalis Excavatio rectovesicalis

Excavatio vesicouterina Excavatio vesicouterina

Excavatio rectouterina Excavatio rectouterina

13 / 200
Щуру в плевральну порожнину введено 0,5 мл повітря. Який тип недостатності дихання виникає в даному випадку? 0.5 ml of air was injected into the pleural cavity of a rat. What type of respiratory failure occurs in this case?

Дисрегуляторне порушення альвеолярної вентиляції Dysregulatory disorder of alveolar ventilation

Обструктивне порушення альвеолярної вентиляції Obstructive violation of alveolar ventilation

Дифузійний Diffuse

Перфузійний Perfusion

Рестриктивне порушення альвеолярної вентиляції Restrictive violation of alveolar ventilation

14 / 200
Дитина 10-ти років страждає на стафілококовий дерматит. Лікування бензилпеніциліном не дало результатів. Призначення комбінованого препарату пеніциліну з клавулановою кислотою дало швидке одужання. Яка причина позитивної дії цього препарату? A 10-year-old child suffers from staphylococcal dermatitis. Treatment with benzylpenicillin did not give results. The appointment of a combined penicillin with clavulanic acid resulted in a quick recovery. What is the reason for the positive effect of this drug ?

Інактивація β-лактамази Inactivation of β-lactamase

Блокада транслокази Translocase blockade

Активація фосфодіестерази Activation of phosphodiesterase

Гальмування транспептидази Inhibition of transpeptidase

Гальмування аденозиндезамінази Inhibition of adenosine deaminase

15 / 200
У хворого спостерігається типова для нападу малярії клінічна картина: озноб, жар, проливний піт. Яка стадія малярійного плазмодію найімовірніше буде виявлена в крові хворого в цей час? The patient has a clinical picture typical of a malaria attack: chills, fever, profuse sweat. What stage of malaria plasmodium is most likely to be detected in the patient's blood at this time?

Оокінета Ookineta

Мерозоїт Merozoite

Спорозоїт Sporozoite

Спороциста Sporocyst

Мікро- або макрогамети Micro or macro gametes

16 / 200
У немовляти спостерігаються епілептиформні судоми, викликані дефіцитом вітаміну В6. Це спричинено зменшенням у нервовій тканині гальмівного медіатора - γ-аміномасляної кислоти. Активність якого ферменту знижена при цьому? A baby has epileptiform seizures caused by a deficiency of vitamin B6. This is caused by a decrease in the nerve tissue of the inhibitory mediator - γ-aminobutyric acid. The activity of which enzyme is reduced?

Глутаматдекарбоксилаза Glutamate decarboxylase

Піридоксалькіназа Pyridoxal Kinase

Глутаматдегідрогеназа Glutamate dehydrogenase

Аланінамінотрансфераза Alanine aminotransferase

Глутаматсинтетаза Glutamate synthetase

17 / 200
Жінку 44-х років вжалила оса внаслідок чого розвинувся шок. В анамнезі вже була важка алергічна реакція на жалення оси. Об’єктивно: пульс -179/хв., слабкий, АТ- 80/40 ммрт.ст., ЧД-26/хв. Яка провідна ланка патогенезу анафілактичного шоку? A 44-year-old woman was stung by a wasp, as a result of which she developed shock. She already had a severe allergic reaction to a wasp sting in the anamnesis. Objectively: pulse -179/min. , weak, blood pressure - 80/40 mmHg, blood pressure - 26/min. What is the leading link in the pathogenesis of anaphylactic shock?

Зменшення ударного об’єму серця Decreased stroke volume of the heart

Зменшення об’єму циркулюючої крові Decreasing the volume of circulating blood

Зниження периферійного опору судин Reduction of peripheral vascular resistance

Біль Pain

Тахікардія Tachycardia

18 / 200
У препараті в одній з судин мікроциркуляторного русла середня оболонка утворена 1-2 шарами гладеньких міоцитів, які розташовані поодинці і мають спіралеподібний напрямок. Зовнішня оболонка представлена тонким шаром пухкої волокнистої сполучної тканини. Вкажіть вид судини: In the preparation, in one of the vessels of the microcirculatory bed, the middle membrane is formed by 1-2 layers of smooth myocytes, which are located alone and have a spiral direction. The outer membrane is represented by a thin layer of loose fibrous connective tissue. Specify the type of vessel:

Артеріоловенулярний анастомоз Arteriovenular anastomosis

Капіляр Capillary

Артеріола Arteriole

Венула Venula

Посткапіляр Postcapillary

19 / 200
Фекалії дитини, що хворіє на ентерит, емульгують в фізіологічному розчині і краплю емульсії наносять на елективне середовище: 10% молочно-сольовий, або жовтково-сольовий агар. Який мікроорганізм передбачається виділити? Feces of a child suffering from enteritis are emulsified in a physiological solution and a drop of the emulsion is applied to an elective medium: 10% milk-salt or yolk-salt agar. Which the microorganism is supposed to be isolated?

Ентерокок Enterococcus

Клебсієла Klebsiella

Кишкова паличка Escherichia coli

Стафілокок Staphylococcus

Стрептокок Streptococcus

20 / 200
При обстеженні хворого виявлена характерна клініка колагенозу Вкажіть, збільшення якого показника сечі характерне для цієї патології: During the examination of the patient, a characteristic collagenosis clinic was revealed. Indicate which increase in the urine indicator is characteristic of this pathology:

Солі амонію Ammonium salts

Аргінін Arginine

Гідроксипролін Hydroxyproline

Мінеральні солі Mineral salts

Глюкоза Glucose

21 / 200
При патологічних процесах, які супроводжуються гіпоксією, відбувається неповне відновлення молекули кисню в дихальному ланцюзі і накопичення пероксиду водню. Вкажіть фермент, який забезпечує його руйнування: In pathological processes that are accompanied by hypoxia, there is an incomplete restoration of the oxygen molecule in the respiratory chain and the accumulation of hydrogen peroxide. Specify the enzyme that ensures its destruction:

Кетоглутаратдегідрогеназа Ketoglutarate dehydrogenase

Сукцинатдегідрогеназа Succinate dehydrogenase

Цитохромоксидаза Cytochrome oxidase

Каталаза Catalase

Аконітаза Aconitase

22 / 200
Електрофоретичне дослідження сироватки крові хворого пневмонією показало збільшення одної з білкових фракцій. Вкажіть її: Electrophoretic examination of the blood serum of a pneumonia patient showed an increase in one of the protein fractions. Specify it:

Альбуміни Albumins

β-глобуліни β-globulins

α2 -глобуліни α2 -globulins

α1 -глобуліни α1 -globulins

γ-глобуліни γ-globulins

23 / 200
При травмі в області тазу у хворого на рентгенологічному знімку виявлено некроз головки стегнової кістки. Під час травми кульшового суглоба було пошкоджено зв’язку: With an injury in the pelvic area, necrosis of the femoral head was detected in the patient's X-ray image. During the injury of the hip joint, the ligament was damaged:

Сіднично-стегнова Gluteal-femoral

Клубово-стегнова Iliofemoral

Лобкова-стегнова pubic-femoral

Головки стегнової кістки Femoral heads

- -

24 / 200
Людині внутрішньовенно ввели 0,5 л ізотонічного розчину лікарської речовини. Які з рецепторів насамперед прореагують на зміни водно-сольового балансу організму? A person intravenously injected 0.5 l of an isotonic solution of a medicinal substance. Which of the receptors will first of all react to changes in the body's water-salt balance?

Натрієві рецептори гіпоталамуса Sodium receptors of the hypothalamus

Осморецептори гіпоталамусу Osmoreceptors of the hypothalamus

Осморецептори печінки Osmoreceptors of the liver

Барорецептори дуги аорти Baroreceptors of the aortic arch

Волюморецептори порожнистих вен і передсердь Volume receptors of the vena cava and atria

25 / 200
В судово-медичній експертизі широко використовується метод дактилоскопії, який оснований на тому, що сосочковий шар дерми визначає строго індивідуальний малюнок на поверхні шкіри. Яка тканина утворює цей шар дерми? The dactyloscopy method is widely used in forensic medical examination, which is based on the fact that the papillary layer of the dermis determines a strictly individual pattern on the surface of the skin. What tissue forms this layer of the dermis ?

Ретикулярна тканина Reticular tissue

Жирова тканина Adipose tissue

Щільна оформлена сполучна тканина Dense designed connective tissue

Пухка волокниста неоформлена сполучна частина Fluffy fibrous rough connector

Щільна неоформлена сполучна тканина Dense unformed connective tissue

26 / 200
Важливою складовою частиною ниркового фільтраційного бар’єру є тришарова базальна мембрана, яка має спеціальну сітчасту будову її середнього електроннощільного шару. Де міститься ця базальна мембрана? An important component of the kidney's filtration barrier is the three-layer basement membrane, which has a special mesh structure of its middle electron-dense layer. Where is this basement membrane located?

Ниркове тільце Renal corpuscle

Тонкі канальці Thin tubules

Дистальні прямі канальці Distal straight tubules

Проксимальні канальці Proximal tubules

Капіляри перитубулярної капілярної сітки Capillaries of the peritubular capillary network

27 / 200
Чоловік 42-х років помер при явищах вираженої інтоксикації і дихальної недостатності. На розтині: тканина легень у всіх відділах строката, з множинними дрібновогнищевими крововиливами та вогнищами емфіземи. Гістологічно у легенях: геморагічна бронхопневмонія з абсцедуванням, у цитоплазмі клітин епітелію бронхів еозинофільні і базофільні включення. Діагностуйте виявлене на секції захворювання: A 42-year-old man died due to symptoms of severe intoxication and respiratory failure. At autopsy: lung tissue in all departments is variegated, with multiple small focal hemorrhages and foci of emphysema. Histologically in the lungs: hemorrhagic bronchopneumonia with abscessation, eosinophilic and basophilic inclusions in the cytoplasm of the cells of the bronchial epithelium. Diagnose the disease detected on the section:

Грип Flu

Часткова пневмонія Partial pneumonia

Плевропневмонія Pleuropneumonia

Стафілококова бронхопневмонія Staphylococcal bronchopneumonia

Крупозна пневмонія Croup pneumonia

28 / 200
В експерименті подразнюють скелетний м’яз серією електричних імпульсів. Який вид м’язового скорочення виникне, якщо кожний наступний імпульс припадає на період вкорочення поодинокого м’язового скорочення? In an experiment, a skeletal muscle is stimulated with a series of electrical impulses. What type of muscle contraction will occur if each subsequent impulse falls within the period of shortening of a single muscle contraction?

Суцільний тетанус Solid tetanus

Зубчастий тетанус Serrated tetanus

Контрактура м’яза Muscle contracture

Асинхронний тетанус Asynchronous tetanus

Серія поодиноких скорочень Series of single abbreviations

29 / 200
У чоловіка 53-х років діагностовано сечокам’яну хворобу з утворенням уратів. Цьому пацієнту призначено аллопурінол, який є конкурентним інгібітором ферменту: A 53-year-old man was diagnosed with urolithiasis with urate formation. This patient was prescribed allopurinol, which is a competitive enzyme inhibitor:

Уреаза Urease

Дигідроурацилдегідрогеназа Dihydrouracil dehydrogenase

Уратоксидаза Urate oxidase

Уриділтрансфераза Uridyltransferase

Ксантиноксидаза Xanthine oxidase

30 / 200
Мати зауважила занадто темну сечу у її 5-річної дитини. Дитина скарг не висловлює. Жовчних пігментів у сєчі не виявлено. Поставлено діагноз алкаптонурія. Дефіцит якого ферменту має місце у дитини? The mother noticed too dark urine in her 5-year-old child. The child does not complain. Bile pigments were not detected in the urine. A diagnosis of alkaptonuria was made. What enzyme deficiency occurs does the child have?

Оксидаза оксифенілпірувату Oxyphenylpyruvate oxidase

Оксидаза гомогентизинової кислоти Homogentisic acid oxidase

Фенілаланінгідроксилаза Phenylalanine hydroxylase

Тирозиназа Tyrosinase

Декарбоксилаза фенілпірувату Phenylpyruvate decarboxylase

31 / 200
До дерматолога звернулася пацієнтка із скаргами на екзематозне ураження шкіри рук, що з’являється після контакту з миючим засобом ”Лотос” Використання гумових рукавичок запобігає цьому. Патологічна реакція шкіри зумовлена активацією: A patient turned to a dermatologist with complaints of eczematous damage to the skin of the hands, which appeared after contact with the detergent 'Lotus'. The use of rubber gloves prevents this. Pathological reaction of the skin due to activation:

Нейтрофілів Neutrophils

Т-лімфоцитів T-lymphocytes

В-лімфоцитів B-lymphocytes

Моноцитів Monocytes

Базофілів Basophils

32 / 200
У хворої 36-ти років, яка лікувалася сульфаніламідами з приводу респіраторної вірусної інфекції, в крові гіпорегенераторна нормохромна анемія, лейкопенія, тромбоцитопенія. В кістковому мозку - зменшення кількості мієлокаріоцитів. Яка це анемія? A 36-year-old patient, who was treated with sulfonamides for a respiratory viral infection, has hyporegenerative normochromic anemia, leukopenia, thrombocytopenia in the blood. In the bone marrow - a decrease in the number of myelokaryocytes . What kind of anemia is this?

Залізодефіцитна Iron deficiency

В12-фолієводефіцитна B12-folate deficient

Гіпопластична Hypoplastic

Постгеморагічна Posthemorrhagic

Гемолітична Hemolytic

33 / 200
Чоловік 38-ми років раптово помер. На розтині: у задній стінці лівого шлуночка серця виявлено інфаркт міокарда. Які найбільш імовірні зміни у будові міокардіоцитів можна побачити у вогнищі інфаркту мікроскопічно? A 38-year-old man died suddenly. At autopsy: a myocardial infarction was found in the back wall of the left ventricle of the heart. What are the most likely changes in the structure of myocardiocytes that can be seen in the heart of the infarction microscopically?

Звапнування Calcification

Білкова дистрофія Protein dystrophy

Жирова дистрофія Fat dystrophy

Вуглеводна дистрофія Carbohydrate dystrophy

Каріолізис Karyolysis

34 / 200
У відділення реанімації надійшов чоловік 47-ми років з діагнозом інфаркт міокарда. Яка з фракцій лактатдегідрогенази (ЛДГ) буде переважати в сироватці крові впродовж перших двох діб захворювання? A 47-year-old man was admitted to the intensive care unit with a diagnosis of myocardial infarction. Which fraction of lactate dehydrogenase (LDH) will predominate in the blood serum during the first two days of the disease?

ЛДГ1 LDH1

ЛДГ2 LDH2

ЛДГ5 LDH5

ЛДГ3 LDH3

ЛДГ4 LDH4

35 / 200
У хлопчика 2-х років спостерігається збільшення в розмірах печінки та селезінки, катаракта. В крові підвищена концентрація цукру, але тест толерантності до глюкози в нормі. Спадкове порушення обміну якої речовини є причиною цього стану? A 2-year-old boy has an increase in the size of the liver and spleen, a cataract. The concentration of sugar in the blood is increased, but the glucose tolerance test is normal. Hereditary metabolic disorder what substance is the cause of this condition?

Сахароза Sucrose

Фруктоза Fructose

Мальтоза Maltose

Галактоза Galactose

Глюкоза Glucose

36 / 200
У неврологічне відділення з приводу мозкового крововиливу поступив хворий 62-х років. Об’єктивно: стан важкий. Спостерігається наростання глибини і частоти дихання, а потім його зменшення до апное, після чого цикл дихальних рухів відновлюється. Який тип дихання у хворого? A 62-year-old patient was admitted to the neurology department due to cerebral hemorrhage. Objectively: the condition is severe. An increase in the depth and frequency of breathing is observed, and then it decreases to apnea, after which the cycle of respiratory movements is restored. What type of breathing does the patient have?

Апнейстичне Apneistic

Чейна-Стокса Cheyne-Stokes

Біота Biota

Кусмауля Kusmaul

Гаспінг-дихання Gasping Breath

37 / 200
Глікоген, що надійшов з їжею, гідролізується у шлунково-кишковому тракті. Який кінцевий продукт утворюється в результаті цього процесу? Dietary glycogen is hydrolyzed in the gastrointestinal tract. What is the end product of this process?

Глюкоза Glucose

Лактоза Lactose

Галактоза Galactose

Лактат Lactate

Фруктоза Fructose

38 / 200
Після перенесеного геморагічного інсульту у хворого розвинулася кіста головного мозку. Через 2 роки помер від післягрипозної пневмонії. На секції трупа виявлено в мозку кісту із стінками біло-іржавого відтінку, реакція Перлса позитивна. Який з процесів найбільш імовірний у стінці кісти? After a hemorrhagic stroke, the patient developed a brain cyst. 2 years later, he died of post-influenza pneumonia. An autopsy revealed a brain cyst with white-rusty walls, the Perls reaction is positive. Which of the processes is most likely in the cyst wall?

Місцевий гемосидероз Local hemosiderosis

Інфільтрація білірубіну Bilirubin infiltration

Первинний гемохроматоз Primary hemochromatosis

Місцевий гемомеланоз Local hemomelanosis

Загальний гемосидероз General hemosiderosis

39 / 200
При розтині тіла померлого чоловіка 73-х років, який довго страждав на ішемічну хворобу серця з серцевою недостатністю, знайдено: ’’мускатна” печінка, бура індурація легень, ціанотична індурація нирок та селезінки. Який з видів порушення кровообігу найбільш імовірний? At the autopsy of the dead man of 73, who suffered from coronary heart disease with heart failure for a long time, the following were found: 'nutmeg' liver, brown induration of the lungs, cyanotic induration of the kidneys and spleen. Which type of circulatory disturbance is most likely?

Гостре загальне венозне повнокрів’я Acute general venous congestion

Хронічне загальне венозне повнокрів’я Chronic general venous congestion

Гостре малокрів’я Acute anemia

Артеріальна гіперемія Arterial hyperemia

Хронічне малокрів’я Chronic anemia

40 / 200
У нейрохірургічне відділення поступив 54-річний чоловік із скаргами на відсутність чутливості шкіри нижньої повіки, латеральної поверхні зовнішнього носа, верхньої губи. Лікар при огляді встановлює запалення другої гілки трійчастого нерва. Через який отвір виходить із черепа ця гілка? A 54-year-old man was admitted to the neurosurgery department with complaints of lack of sensitivity of the skin of the lower eyelid, the lateral surface of the external nose, and the upper lip. During the examination, the doctor found inflammation of the second branch of the trigeminal nerve. Through which hole does this branch leave the skull?

Остистий отвір Prickly Hole

Верхня очноямкова щілина Upper orbital fissure

Рваний отвір Torn hole

Овальний отвір Oval hole

Круглий отвір Round hole

41 / 200
Чоловіку 18-ти років з приводу флегмони плеча було зроблено внутрішньом’язову ін’єкцію пеніциліну. Після цього у нього з’явилися тахікардія, ниткоподібний пульс, АТ знизився до 80/60 мм рт.ст. Який вид фармакологічної реакції розвинувся? An 18-year-old man was given an intramuscular injection of penicillin due to phlegmon of the shoulder. After that, he developed tachycardia, a threadlike pulse, and blood pressure decreased up to 80/60 mm Hg. What type of pharmacological reaction developed?

Периферична дія Peripheral action

Анафілаксія Anaphylaxis

Центральна дія Central Action

Рефлекторна дія Reflex action

Потенціювання Potentialization

42 / 200
Після побутової травми у пацієнта 18-ти років з’явились постійні запаморочення, ністагм очей, скандована мова, невпевнена хода. Це свідчить про порушення функції: After a domestic injury, an 18-year-old patient developed constant dizziness, eye nystagmus, slurred speech, and unsteady gait. This indicates a functional impairment:

Вестибулярних ядер Vestibular nuclei

Базальних гангліїв Basal ganglia

Чорної субстанції Black substance

Рухової кори Motor cortex

Мозочка Cerebellum

43 / 200
Хворий 65-ти років, що страждає на атеросклероз, госпіталізований до хірургічного відділення з приводу розлитого гнійного перитоніту. Під час операції діагностовано тромбоз брижових артерій. Яка найбільш імовірна причина перитоніту? A 65-year-old patient suffering from atherosclerosis was admitted to the surgical department for diffuse purulent peritonitis. During the operation, thrombosis of the mesenteric arteries was diagnosed. What is the most likely cause peritonitis?

Стаз Stage

Ішемічний інфаркт Ischemic heart attack

Ішемія компресійна Compression ischemia

Геморагічний інфаркт Hemorrhagic heart attack

Ішемія ангіоспастична Angiospastic ischemia

44 / 200
Хворому з прогресуючою м’язовою дистрофією було проведено біохімічне дослідження сечі. Поява якої речовини у великій кількості в сечі може підтвердити захворювання м’язів у даного хворого? A patient with progressive muscular dystrophy underwent a biochemical examination of urine. The appearance of a substance in large quantities in the urine can confirm muscle disease in this patient?

Сечовина Urea

Порфірини Porphyrins

Гіпурова кислота Hippuric acid

Креатинін Creatinine

Креатин Creatine

45 / 200
У крові чоловіка 26-ти років виявлено 18% еритроцитів сферичної, сплощеної, кулястої та остистої форм. Інші еритроцити були у формі двоввігнутих дисків. Як називається таке явище? In the blood of a 26-year-old man, 18% of erythrocytes were found to be spherical, flattened, spherical and spiny. Other erythrocytes were in the form of biconcave discs. What is this phenomenon called?

Фізіологічний анізоцитоз Physiological anisocytosis

Патологічний пойкілоцитоз Pathological poikilocytosis

Патологічний анізоцитоз Pathological anisocytosis

Еритроцитоз Erythrocytosis

Фізіологічний пойкілоцитоз Physiological poikilocytosis

46 / 200
Хвора страждає від болю в ногах та набряків. В ході обстеження хворої на медіальній поверхні стегна спостерігається набряк, збільшення розміру вен, утворення вузлів. З боку якої вени спостерігається патологія? The patient suffers from pain in the legs and swelling. During the examination of the patient on the medial surface of the thigh, there is swelling, an increase in the size of the veins, and the formation of nodes. On which side of the vein is the pathology observed ?

V tibialis V tibialis

V saphena magna V saphena magna

V saphena parva V saphena parva

V profunda femoris V profunda femoris

V femoralis V femoralis

47 / 200
Під час підготовки пацієнта до операції на серці проведено вимірювання тиску в камерах серця. В одній з них тиск протягом серцевого циклу змінювався від 0 до 120 мм рт.ст. Назвіть цю камеру серця: During the patient's preparation for heart surgery, pressure was measured in the chambers of the heart. In one of them, the pressure during the cardiac cycle varied from 0 to 120 mmHg. Name this chamber of the heart:

- -

Ліве передсердя Left atrium

Правий шлуночок Right ventricle

Праве передсердя Right atrium

Лівий шлуночок Left Ventricle

48 / 200
У жінки обмежений кровотік у нирках, підвищений артеріальний тиск. Гіперсекреція якого гормону зумовила підвищений тиск? The woman has limited blood flow in the kidneys, high blood pressure. Hypersecretion of which hormone caused the high pressure?

Ренін Renin

Норадреналін Noradrenaline

Вазопресин Vasopressin

Еритропоетин Erythropoietin

Адреналін Adrenaline

49 / 200
На останньому місяці вагітності вміст фібриногену в плазмі крові в 2 рази вище за норму. Яку швидкість осідання еритроцитів слід при цьому очікувати? In the last month of pregnancy, the content of fibrinogen in the blood plasma is 2 times higher than the norm. What should be the rate of erythrocyte sedimentation?

10-15 мм/годину 10-15 mm/hour

40-50 мм/годину 40-50 mm/hour

5-10 мм/годину 5-10 mm/hour

3-12 мм/годину 3-12 mm/hour

0-5 мм/годину 0-5 mm/hour

50 / 200
Пацієнт 16-ти років, що страждає на хворобу Іценко-Кушінга, консультований з приводу надмірної ваги тіла. При опитуванні з’ясувалося, що енергетична цінність спожитої їжі складає 1700-1900 ккал/добу. Яка провідна причина ожиріння у даному випадку? A 16-year-old patient suffering from Itsenko-Cushing's disease is consulted about excess body weight. The survey revealed that the energy value of the consumed food is 1700-1900 kcal/day. What is the leading cause of obesity in this case?

Нестача інсуліну Lack of insulin

Надлишок глюкокортикоїдів Glucocorticoid excess

Нестача глюкокортикоїдів Lack of glucocorticoids

Гіподинамія Hypodynamia

Надлишок інсуліну Excess insulin

51 / 200
Людина зробила спокійний видих. Як називається об’єм повітря, який міститься у неї в легенях при цьому? A person exhaled calmly. What is the volume of air contained in his lungs during this called?

Резервний об’єм видиху Reserve expiratory volume

Дихальний об’єм Respiratory volume

Функціональна залишкова ємкість легень Functional residual lung capacity

Життєва ємність легень Vital lung capacity

Залишковий об’єм Remaining Volume

52 / 200
Людина зробила максимально глибокий видих. Як називається об’єм повітря, що знаходиться в її легенях після цього? A person exhaled as deeply as possible. What is the volume of air in his lungs after that called?

Залишковий об’єм Remaining Volume

Резервний об’єм видиху Reserve expiratory volume

Ємність вдиху Inhalation capacity

Альвеолярний об’єм Alveolar volume

Функціональна залишкова ємність легень Functional residual lung capacity

53 / 200
В експерименті на ссавці зруйнували певну структуру серця, що призвело до припинення проведення збудження від передсердь до шлуночків. Що саме зруйнували? In an experiment on a mammal, a certain structure of the heart was destroyed, which led to the cessation of conduction of excitation from the atria to the ventricles. What exactly was destroyed?

Пучок Гіса His Bundle

Ніжки пучка Гіса His Bundle Legs

Синоатріальний вузол Sinoatrial node

Волокна Пуркін’є Purkinje fibers

Атріовентрикулярний вузол Atrioventricular node

54 / 200
Необхідно оцінити рівень збудливості нерва у хворого. Для цього доцільно визначити для нерва наступну величину: It is necessary to assess the level of nerve excitability in the patient. For this, it is advisable to determine the following value for the nerve:

Тривалість потенціалу дії Action potential duration

Критичний рівень деполяризації Critical level of depolarization

Порогова сила подразника Threshold stimulus strength

Потенціал спокою Rest potential

Амплітуда потенціалу дії Action potential amplitude

55 / 200
Хворому з гострою недостатністю надниркових залоз був призначений лікарський препарат після якого у нього з’явилися скарги на біль в кістках (двічі були переломи), часті простудні хвороби, набряки, повільне загоювання ран. Який препарат міг спричинити такі явища? A patient with acute adrenal insufficiency was prescribed a drug, after which he complained of pain in the bones (twice had fractures), frequent colds, swelling , slow wound healing. What drug could cause such phenomena?

Естріол Estriol

Преднізолон Prednisone

Тестостерон Testosterone

Ретаболіл Retabolil

Спіронолактон Spironolactone

56 / 200
Під час розтину тіла мертвонародженої дитини виявлено аномалію розвитку серця: шлуночки не розмежовані, з правої частини виходить суцільний артеріальний стовбур. Для яких хребетних характерна подібна будова серця? During the autopsy of the body of a stillborn child, an abnormality in the development of the heart was found: the ventricles are not demarcated, a solid arterial trunk emerges from the right part. What vertebrates have a similar structure of the heart?

Амфібії Amphibia

Риби Fish

Рептилії Reptiles

Птахи Birds

Ссавці Mammals

57 / 200
Який з нижченаведених сечогінних засобів слід призначити хворому з первинним гіперальдостеронізмом? Which of the following diuretics should be prescribed to a patient with primary hyperaldosteronism?

Маніт Manit

Триамтерен Triamterene

Спіронолактон Spironolactone

Гіпотіазид Hypotiazid

Фуросемід Furosemide

58 / 200
В результаті травми хворий не може підняти руку до горизонтального рівня. Який м’яз постраждав? As a result of the injury, the patient cannot raise his arm to the horizontal level. What muscle is affected?

Двоголовий Two-headed

Дельтоподібний Delta

Найширший м’яз спини Latisus dorsi

Трапецієподібний Trapezoid

Триголовий Three-headed

59 / 200
В експерименті збільшили проникність мембрани збудливої клітини для іонів калію. Які зміни електричного стану мембрани при цьому виникнуть? In the experiment, the permeability of the excitable cell membrane for potassium ions was increased. What changes in the electrical state of the membrane will occur?

Деполяризація Depolarization

Потенціал дії Action potential

Локальна відповідь Local response

Гіперполяризація Hyperpolarization

Змін не буде There will be no changes

60 / 200
При обстеженні в аналізі крові пацієнта виявлено лейкоцитоз, лімфоцитоз, клітини Боткіна- Гумпрехта на тлі анемії. Про яку хворобу слід думати лікарю? When examining the patient's blood, leukocytosis, lymphocytosis, Botkin-Gumprecht cells against the background of anemia were detected. What disease should the doctor think about?

Мієломна хвороба Myeloma

Гострий мієлолейкоз Acute myelogenous leukemia

Хронічний лімфолейкоз Chronic lymphocytic leukemia

Інфекційний мононуклеоз Infectious mononucleosis

Лімфогранулематоз Lymphogranulomatosis

61 / 200
У жінки 37-ми років протягом року періодично виникали інфекційні захворювання бактеріального генезу, їх перебіг був вкрай тривалим, ремісії - короткочасними. При обстеженні виявлена гіпогамаглобулінемія. Порушення функції яких клітин може бути прямою її причиною? A 37-year-old woman periodically developed infectious diseases of bacterial origin during the year, their course was extremely long, remissions were short-lived. During the examination, hypogammaglobulinemia was detected. Violations of the functions of cells can be its direct cause?

Макрофаги Macrophages

Фагоцити Phagocytes

Лімфоцити Lymphocytes

Плазматичні клітини Plasma cells

Нейтрофіли Neutrophils

62 / 200
У хворого 37-ми років на фоні тривалого застосування антибіотиків спостерігається підвищена кровоточивість при невеликих пошкодженнях. У крові - зниження активності факторів згортання крові ІІ, VII, IX, X, подовження часу згортання крові. Недостатністю якого вітаміну обумовлені зазначені зміни? In a 37-year-old patient, against the background of long-term use of antibiotics, there is increased bleeding with small injuries. In the blood, there is a decrease in the activity of blood coagulation factors II, VII, IX, X , lengthening of the blood clotting time. The deficiency of which vitamin is responsible for these changes?

Вітамін К Vitamin K

Вітамін А Vitamin A

Вітамін Е Vitamin E

Вітамін С Vitamin C

Вітамін D Vitamin D

63 / 200
Хворий 18-ти років звернувся до лікарні із скаргами на шум та больові відчуття у вусі. Об’єктивно - у хворого гостре респіраторне захворювання, риніт. Крізь який отвір глотки інфекція потрапила до барабанної порожнини та викликала її запалення? An 18-year-old patient came to the hospital with complaints of noise and pain in the ear. Objectively, the patient has an acute respiratory disease, rhinitis. Through which opening pharynx infection got to the tympanic cavity and caused its inflammation?

Зів Ziv

Вхід до гортані Entrance to the larynx

Глотковий отвір слухової труби Pharyngeal opening of the auditory tube

Хоани Hoans

Барабанний отвір слухової труби Tympanic opening of the auditory tube

64 / 200
У хворого 68-ми років, який страждає на серцеву недостатність та впродовж тривалого часу приймає препарати наперстянки, з’явилися явища інтоксикації, які швидко нівелювалися застосуванням донатора сульфгідрильних груп унітіолу. Який механізм терапевтичної дії цього засобу? In a 68-year-old patient who suffers from heart failure and has been taking digitalis drugs for a long time, intoxication symptoms appeared, which were quickly eliminated by the use of a donor of sulfhydryl groups unitiol. What is the mechanism of therapeutic action of this agent?

Зменшує накопичення іонізованого кальцію Reduces the accumulation of ionized calcium

Підвищує енергозабезпечення міокарду Increases the energy supply of the myocardium

Іальмує вивільнення калію з міокардіоцитів Increases potassium release from myocardiocytes

Сповільнює надходження натрію до міокардіоцитів Slows sodium intake to myocardiocytes

Реактивує натрій-калієву-АТФ-азу мембран міокардіоцитів Reactivates sodium-potassium-ATP-ase of myocardiocyte membranes

65 / 200
Охолодження тіла людини у воді виникає значно швидше, ніж на повітрі, тому, що у воді значно ефективнішою є віддача тепла шляхом: Cooling of the human body occurs much faster in water than in air, because heat transfer is much more efficient in water by:

Конвекції Convection

- -

Тепловипромінювання Heat radiation

Теплопроведення Heat conduction

Випаровування поту Evaporation of sweat

66 / 200
У бактеріологічній лабораторії проводиться дослідження якості питної води. Її мікробне число виявилося близько 100. Які мікроорганізми враховувалися при цьому? The quality of drinking water is being tested in the bacteriological laboratory. Its microbial count was found to be about 100. What microorganisms were taken into account?

Умовно-патогенні мікроорганізми Conditionally pathogenic microorganisms

Всі бактерії, що виросли на живильному середовищі All bacteria grown on nutrient medium

Бактерії, патогенні для людей та тварин Bacteria pathogenic for humans and animals

Ентеропатогенні бактерії та віруси Enteropathogenic bacteria and viruses

Бактерії групи кишкової палички Escherichia coli bacteria

67 / 200
У юнака 20-ти років діагностовано спадковий дефіцит УДФ-глюкуронілтрансферази. Підвищення якого показника крові підтверджує діагноз? A 20-year-old man was diagnosed with a hereditary deficiency of UDP-glucuronyltransferase. An increase in which blood level confirms the diagnosis?

Уробілін Urobilin

Непрямий (некон’югований) білірубін Indirect (unconjugated) bilirubin

Стеркобіліноген Stercobilinogen

Прямий (кон’югований) білірубін Direct (conjugated) bilirubin

Тваринний індикан Animal Indican

68 / 200
Хворому 50-ти років з хронічною серцевою недостатністю і тахиаритмією призначили кардіотонічний препарат. Який з препаратів призначили хворому? A 50-year-old patient with chronic heart failure and tachyarrhythmia was prescribed a cardiotonic drug. Which drug was prescribed to the patient?

Добутамін Dobutamine

Дофамін Dopamine

Дигоксин Digoxin

Мілдронат Mildronate

Аміодарон Amiodarone

69 / 200
У людини порушено всмоктування продуктів гідролізу жирів. Причиною цього може бути дефіцит у порожнині тонкої кишки: A person has impaired absorption of fat hydrolysis products. The reason for this may be a deficiency in the cavity of the small intestine:

Ліполітичних ферментів Lipolytic enzymes

Жовчних пігментів Bile pigments

Іонів натрію Sodium ions

Жовчних кислот Bile acids

Жиророзчинних вітамінів Fat-soluble vitamins

70 / 200
У відповідь на сильне швидке скорочення м’яза спостерігається його рефлекторне розслаблення. З подразнення яких рецепторів починається ця рефлекторна реакція? In response to a strong rapid contraction of a muscle, its reflex relaxation is observed. With the irritation of which receptors does this reflex reaction begin?

Больові рецептори Pain receptors

Дотикові рецептори Touch receptors

Суглобові рецептори Joint receptors

Сухожилкові рецептори Гольджі Golgi tendon receptors

М’язові веретена Muscle spindles

71 / 200
У хворого, що переніс 5 років тому субтотальну резекцію шлунка, розвинулась В12-фолієводефіцитна анемія. Який механізм є провідним у розвитку такої анемії? A patient who underwent a subtotal resection of the stomach 5 years ago developed B12-folate-deficient anemia. What is the leading mechanism in the development of such anemia?

Відсутність зовнішнього фактора Касла Absence of external Castle factor

Дефіцит фолієвої кислоти Folic acid deficiency

Порушення всмоктування вітаміну В в тонкій кишці Disturbance of vitamin B absorption in the small intestine

Дефіцит транскобаламіну Transcobalamin deficiency

Відсутність внутрішнього фактора Касла No internal Castle factor

72 / 200
Чоловік 50-ти років хворіє на хронічний бронхіт, скаржиться на задишку під час фізичного навантаження, постійний кашель з відходженням харкотиння. При обстеженні діагностовано ускладнення - емфізема легень. Чим вона зумовлена? A 50-year-old man suffers from chronic bronchitis, complains of shortness of breath during physical exertion, constant cough with expectoration. During the examination, a complication was diagnosed - emphysema of the lungs. What is it conditioned?

Зменшення альвеолярної вентиляції Decreased alveolar ventilation

Зменшення перфузії легень Decreased lung perfusion

Зниження еластичних властивостей легень Decreasing the elastic properties of the lungs

Порушення вентиляційно-перфузійного співвідношення в легенях Violation of the ventilation-perfusion ratio in the lungs

Зменшення розтяжності легень Decreased lung distensibility

73 / 200
При розтині трупа чоловіка 47-ми років, померлого раптово, в інтимі черевного відділу аорти знайдені осередки жовтого кольору у вигляді плям та смуг, що не вибухають над поверхнею інтими. При фарбуванні суданом III спостерігається жовтогаряче забарвлення. Для якої стадії атеросклерозу характерні такі зміни? At the autopsy of a 47-year-old man who died suddenly, yellow cells were found in the intima of the abdominal part of the aorta in the form of spots and stripes that did not explode above the intima surface When stained with Sudan III, a yellow-red color is observed. What stage of atherosclerosis is characterized by such changes?

Ліпоїдозу Lipoidosis

Ліпосклерозу Liposclerosis

Атероматозу Atheromatosis

Атерокальцинозу Atherocalcinosis

Стадія утворення атероматозної виразки Stage of formation of atheromatous ulcer

74 / 200
У клініку госпіталізовано хворого з діагнозом карциноїду кишечника. Аналіз виявив підвищену продукцію серотоніну, який утворюється з амінокислоти триптофан. Який біохімічний механізм лежить в основі даного процесу? A patient with a diagnosis of intestinal carcinoid was hospitalized in the clinic. The analysis revealed an increased production of serotonin, which is formed from the amino acid tryptophan. What biochemical mechanism is the basis of this process?

Декарбоксилювання Decarboxylation

Мікросомальне окиснення Microsomal oxidation

Трансамінування Transamination

Дезамінування Demining

Утворення парних сполук Formation of paired compounds

75 / 200
У батьків, хворих на гемоглобінопатію (аутосомно-домінантний тип успадкування), народилася здорова дівчинка. Які генотипи батьків? Parents suffering from hemoglobinopathy (autosomal dominant type of inheritance) had a healthy girl. What are the parents' genotypes?

Обоє гетерозиготні за геном гемоглобінопатії Both are heterozygous for the hemoglobinopathy gene

Обоє гомозиготні за геном гемоглобінопатії Both are homozygous for the hemoglobinopathy gene

Батько гетерозиготний за геном гемоглобінопатії, у матері цей ген відсутній The father is heterozygous for the hemoglobinopathy gene, the mother does not have this gene

Мати гетерозиготна за геном гемоглобінопатії, у батька цей ген відсутній The mother is heterozygous for the hemoglobinopathy gene, the father lacks this gene

У обох батьків ген гемоглобінопатії відсутній Both parents have no hemoglobinopathy gene

76 / 200
Хворому з артеріальною гіпертензією було призначено один з антигіпертензивних засобів. Артеріальний тиск нормалізувався, однак хворого почав турбувати постійний сухий кашель. Який з перерахованих препаратів має таку побічну дію? A patient with arterial hypertension was prescribed one of the antihypertensive drugs. Blood pressure normalized, but the patient began to be bothered by a constant dry cough. Which of the listed drugs has such a side effect?

Ніфедипін Nifedipine

Резерпін Reserpin

Анаприлін Anaprilin

Лізиноприл Lisinopril

Клофелін Clofelin

77 / 200
В анотації до препарату вказано, що він містить антигени збудника черевного тифу, адсорбовані на стабілізованих еритроцитах барана. З якою метою використовують цей препарат? In the annotation to the drug, it is indicated that it contains antigens of the causative agent of typhoid, adsorbed on stabilized erythrocytes of a ram. For what purpose is this drug used?

Для виявлення антитіл в реакції гальмування гемаглютинації To detect antibodies in the hemagglutination inhibition reaction

Для виявлення антитіл в реакції зв’язування комплементу To detect antibodies in the complement binding reaction

Для виявлення антитіл в реакції Відаля To detect antibodies in the Vidal reaction

Для серологічної ідентифікації збудника черевного тифу For serological identification of the causative agent of typhoid fever

Для виявлення антитіл в реакції непрямої гемаглютинації To detect antibodies in the indirect hemagglutination reaction

78 / 200
При розтині тіла померлого чоловіка 48-ми років в ділянці 1-го сегменту правої легені виявлено круглий утвір діаметром 5 см з чіткими контурами, оточений тонким прошарком сполучної тканини, виповнений білими крихкими масами. Діагностуйте форму вторинного туберкульозу: At the autopsy of the body of a deceased 48-year-old man, a round formation with a diameter of 5 cm with clear contours, surrounded by a thin layer of connective tissue, was found in the area of the 1st segment of the right lung. filled with white friable masses. Diagnose the form of secondary tuberculosis:

Туберкулома Tuberculosis

Казеозна пневмонія Case pneumonia

Гострий кавернозний туберкульоз Acute cavernous tuberculosis

Гострий вогнищевий туберкульоз Acute focal tuberculosis

Фіброзно-кавернозний туберкульоз Fibro-cavernous tuberculosis

79 / 200
У хворого діагностовано септичний ендокардит. Температура тіла протягом 5-ти днів коливалася в межах 39,5oC -40,2oC. На 6-й день на тлі різкого зниження температури до35,2oC розвинувся колапс. Який головний механізм колапсу? The patient was diagnosed with septic endocarditis. The body temperature fluctuated between 39.5oC and 40.2oC for 5 days. On the 6th day, against the background of a sharp decrease at temperatures up to 35.2oC, collapse developed. What is the main mechanism of collapse?

Вазодилатація Vasodilation

Тахікардія Tachycardia

Посилене потовиділення Increased sweating

Гіпервентиляція Hyperventilation

Поліурія Polyuria

80 / 200
Хвора звернулася зі скаргами на болі у правій латеральній ділянці живота. Під час пальпації визначається щільне, нерухоме, пухлиноподібне утворення. У ділянці якого відділу травної трубки можлива наявність пухлини? The patient complained of pain in the right lateral part of the abdomen. During palpation, a dense, immobile, tumor-like formation is detected. In which part of the digestive tube is the presence of a tumor possible?'

Caecum Caecum

Colon sigmoideum Colon sigmoideum

Colon descendens Colon descendens

Colon ascendens Colon ascendens

Colon transversum Colon transversum

81 / 200
У хворого струс головного мозку, що супроводжується повторним блюванням і задишкою. При обстеженні відзначено: рН - 7,62; pCO - 40 мм рт.ст. Яке порушення кислотно-основного стану є у хворого? The patient has a brain concussion, accompanied by repeated vomiting and shortness of breath. During the examination, it was noted: pH - 7.62; pCO - 40 mm Hg. What a violation Does the patient have an acid-base state?

Газовий ацидоз Gas acidosis

Негазовий алкалоз Nongaseous alkalosis

Негазовий ацидоз Nongaseous acidosis

Газовий алкалоз Gas alkalosis

- -

82 / 200
У хворого в обох щелепах рентгенологічно виявлено численні дефекти у вигляді гладкостінних округлих отворів. При гістологічному дослідженні - явища остеолізису і остеопорозу при явищах слабкого кісткоутворення. В сечі хворого знайдено білок Бенс-Джонса. Назвіть захворювання: In both jaws of the patient, numerous defects in the form of smooth-walled round holes were detected x-ray. Histological examination revealed the phenomena of osteolysis and osteoporosis with weak bone formation. Protein was found in the patient's urine Bence-Jones. Name the disease:

Гострий недиференційований лейкоз Acute undifferentiated leukemia

Гострий мієлолейкоз Acute myelogenous leukemia

Хронічний мієлолейкоз Chronic myelogenous leukemia

Хронічний еритромієлоз Chronic erythromyelosis

Мієломна хвороба Myeloma

83 / 200
Катіонні глікопротеїни є основними компонентами слини привушних залоз. Які амінокислоти обумовлюють їх позитивний заряд? Cationic glycoproteins are the main components of parotid gland saliva. What amino acids determine their positive charge?

Аспартат, аргінін, глутамат Aspartate, arginine, glutamate

Глутамат, валін, лейцин Glutamate, valine, leucine

Цистеїн, гліцин, пролін Cysteine, glycine, proline

Аспартат, глутамат, гліцин Aspartate, glutamate, glycine

Лізин, аргінін, гістидин Lysine, arginine, histidine

84 / 200
На мікропрепараті очного яблука плода спостерігається пошкодження рогівки. Частина якого зародкового листка була уражена в процесі ембріонального розвитку? Cornea damage is observed on the micro-preparation of the eyeball of the fetus. Which part of the germ layer was affected during embryonic development?

Ентодерма Entoderm

Мезодерма Mesoderm

Дерматом Dermatoma

Нефротом Nephrotoma

Ектодерма Ectoderm

85 / 200
На практичному занятті з мікробіології студентам запропоновано пофарбувати суміш бактерій за методикою Грама та пояснити механізм фарбування. Які морфологічні структури бактерій зумовлюють грамнегативне та грампозитивне фарбування бактерій? In a practical lesson in microbiology, students were asked to stain a mixture of bacteria using the Gram method and explain the staining mechanism. What morphological structures of bacteria cause gram-negative and gram-positive staining of bacteria?

Джгутики Flags

Цитоплазма Cytoplasm

ЦПМ CPM

Капсула Capsule

Клітинна стінка Cell wall

86 / 200
У вагітної жінки взяли кров для підтвердження клінічного діагнозу 'токсоплазмоз' Яка з перерахованих серологічних реакцій має діагностичне значення? Blood was taken from a pregnant woman to confirm the clinical diagnosis of 'toxoplasmosis'. Which of the listed serological reactions has a diagnostic value?

Реакція гемадсорбції Hemadsorption reaction

Реакція нейтралізації Neutralization reaction

Реакція зв’язування комплементу Complement binding reaction

Реакція аглютинації Agglutination reaction

Реакція гальмування гемаглютинації Hemagglutination inhibition reaction

87 / 200
У здорових батьків, спадковість яких не обтяжена, народилася дитина з множинними вадами розвитку. Цитогенетичний аналіз виявив у соматичних клітинах дитини трисомію за 18-ю хромосомою (синдром Едвардса). З яким явищем пов’язане народження такої дитини? A child with multiple developmental disabilities was born to healthy parents whose heredity is not burdened. Cytogenetic analysis revealed trisomy on the 18th chromosome (Edwards syndrome) in the child's somatic cells What phenomenon is associated with the birth of such a child?

Впливом тератогенних факторів Influence of teratogenic factors

Хромосомною мутацією - дуплікацією By chromosomal mutation - duplication

Домінантною мутацією By dominant mutation

Соматичною мутацією у ембріона By somatic mutation in the embryo

Нерозходженням пари хромосом під час гаметогенезу Non-separation of a pair of chromosomes during gametogenesis

88 / 200
На препараті представлено орган, вкритий сполучнотканинною капсулою, від якої відходять трабекули. В органі можна розрізнити кіркову речовину, де містяться лімфатичні вузлики та мозкову речовину, представлену тяжами лімфоїдних клітин. Який орган представлений на препараті? The preparation shows an organ covered by a connective tissue capsule from which trabeculae depart. In the organ, you can distinguish the cortical substance, which contains lymph nodes, and the medullary substance, represented by strands of lymphoid cells What organ is represented on the drug?

Тимус Thymus

Лімфатичний вузол Lymph node

Мигдалики Tonsils

Селезінка Spleen

Червоний кістковий мозок Red bone marrow

89 / 200
При аналізі родоводу пробанда виявлено, що ознака проявляється з однаковою частотою у представників обох статей і хворі наявні у всіх поколіннях (по вертикалі), а по горизонталі - у сибсів (братів і сестер пробанда) з відносно великих родин. Який тип успадкування досліджуваної ознаки? When analyzing the pedigree of the proband, it was found that the trait appears with the same frequency in representatives of both sexes and patients are present in all generations (vertically), and horizontally - in sibs (siblings of the proband) from relatively large families. What is the type of inheritance of the trait under study?

Зчеплений з Х-хромосомою, домінантний X-linked, dominant

Зчеплений з Х-хромосомою, рецесивний X-linked recessive

Автосомно-рецесивний Autosomal recessive

Зчеплений з Y-хромосомою Linked to Y-chromosome

Автосомно-домінантний Autosomal dominant

90 / 200
У 60-річного пацієнта було виявлено гіперглікемію і глюкозурію. Для лікування хворого лікар призначив препарат для приймання всередину. Який це препарат? A 60-year-old patient was diagnosed with hyperglycemia and glucosuria. To treat the patient, the doctor prescribed a drug to be taken orally. What is this drug?

Фуросемід Furosemide

Панкреатин Pancreatin

Окситоцин Oxytocin

Глібенкламід Glibenclamide

Корглікон Corglycon

91 / 200
Хворий 58-ми років помер від прогресуючої серцевої недостатності. На розтині: серце розширене у поперечнику, мляве, м’яз на розрізі нерівномірного кровонаповнення, пістрявий. При гістологічному дослідженні: у міокарді повнокров’я, у стромі лімфогістіоцитарні інфільтрати, що розсувають кардіоміоцити. Виявлені морфологічні зміни свідчать про: A 58-year-old patient died of progressive heart failure. At autopsy: the heart is dilated in cross section, lethargic, the muscle on the section has uneven blood supply, speckled. At histological in the study: in the myocardium there is blood, in the stroma there are lymphohistiocytic infiltrates, displacing cardiomyocytes. The detected morphological changes indicate:

Кардіосклероз Cardiosclerosis

Жирову дистрофію міокарда Fatty dystrophy of the myocardium

Венозне повнокров’я Venous complete blood

Негнійний проміжний міокардит Nonpurulent interstitial myocarditis

Інфаркт міокарда Myocardial infarction

92 / 200
У важкоатлета при підйомі штанги відбувся розрив грудної лімфатичної протоки. Вкажіть найбільш імовірне місце ушкодження: A weightlifter ruptured a thoracic lymphatic duct while lifting a barbell. Specify the most likely location of the injury:

Ділянка шиї Neck Area

Місце впадіння у венозний кут Place of confluence with the venous angle

Ділянка аортального отвору діафрагми Area of the aortic orifice of the diaphragm

Ділянка попереково-крижового сполучення Section of lumbar-sacral connection

Заднє середостіння Posterior mediastinum

93 / 200
При пошкодженні клітини іонізуючим випромінюванням вмикаються механізми захисту і адаптації. Який механізм відновлення порушеного внутрішньоклітинного гомеостазу реалізується при цьому? When a cell is damaged by ionizing radiation, protection and adaptation mechanisms are activated. What mechanism of restoration of disturbed intracellular homeostasis is implemented in this case?

Пригнічення аденілатциклази Inhibition of adenylate cyclase

Активація антиоксидантної системи Activation of the antioxidant system

Накопичення Na+ в клітинах Na+ accumulation in cells

Активація Са-опосередкованих клітинних функцій Activation of Ca-mediated cellular functions

Гіпертрофія мітохондрій Mitochondrial hypertrophy

94 / 200
У хворої після видалення матки розвинулась гостра анурiя (немає виділення сечі). Які анатомiчнi структури найімовірніше було пошкоджено при операції? After removing the uterus, the patient developed acute anuria (no urine output). What anatomical structures were most likely damaged during the operation?

Внутрішній сфінктер сечівника Internal urethral sphincter

Сечівник Urine

Цибулинно-губчастий м’яз Onion spongy muscle

Сечоводи Ureters

Зовнішній сфінктер сечівника External urethral sphincter

95 / 200
У хворої внаслідок запалення порушена ендокринна функція фолікулярних клітин фолікулів яєчника. Синтез яких гормонів буде пригнічений? The endocrine function of the follicular cells of the ovarian follicles is disturbed in the patient as a result of inflammation. The synthesis of which hormones will be inhibited?

Прогестерон Progesterone

Лютропін Lutropin

Фолістатин Follistatin

Естрогени Estrogens

Фолікулостимулюючий гормон Follicle-stimulating hormone

96 / 200
У чоловіка при обстеженні виявлено порушення кровообігу міокарда лівого передсердя. У басейні якої артерії відбулись порушення кровообігу? During the examination, a man was found to have a violation of the blood circulation of the left atrial myocardium. In the basin of which artery did the blood circulation violations occur?

Права та ліва вінцеві Right and left corona

Ліва вінцева Left coronal

- -

Права вінцева Coronal right

Передня міжшлуночкова гілка лівої вінцевої артерії Anterior interventricular branch of the left coronary artery

97 / 200
Чоловік 36-ти років, лісник за фахом, через тиждень після тривалого перебування у весняному лісі гостро захворів - гарячка, головний біль, порушення свідомості,епілептиформні напади, смерть розвинулася на 3-й день хвороби. На розтині тіла: набрякголовного мозку, множинні точкові геморагії; під час мікроскопічного дослідження - периваскулярний та перицелюлярний набряк, множинні периваскулярні, переважно лімфоцитарні, інфільтрати. Діагностуйте основне захворювання: A 36-year-old man, a forester by profession, became acutely ill a week after a long stay in the spring forest - fever, headache, loss of consciousness, epileptiform seizures, death developed on the 3rd day of illness. On autopsy: cerebral edema, multiple point hemorrhages; during microscopic examination - perivascular and pericellular edema, multiple perivascular, mainly lymphocytic, infiltrates. Diagnose the main disease:

Поліомієліт Polio

Менінгококова інфекція Meningococcal infection

Кліщовий енцефаліт Tick-borne encephalitis

Церебро-васкулярна хвороба Cerebrovascular disease

Гнійний енцефаліт Suppurative encephalitis

98 / 200
При зовнішньому дослідженні трупа чоловіка 69-ти років, який помер 4 години тому, патологоанатом відмітив, що м’язи померлого мають дуже щільну консистенцію, суглоби згинаються та розгинаються важко. Як називається ця патологоанатомічна ознака смерті? During the external examination of the corpse of a 69-year-old man who died 4 hours ago, the pathologist noted that the muscles of the deceased have a very dense consistency, the joints bend and flex difficult. What is the name of this pathological sign of death?

Трупне охолодження Corporal cooling

Трупне заклякання Corpse Charm

Трупні гіпостази Corpse Hypostases

Трупне висихання Corpse desiccation

Трупне розкладання Decomposition

99 / 200
Хворому з метою попередження жирової дистрофії печінки лікар призначив ліпотропний препарат - донор метильних груп. Це імовірно: In order to prevent fatty liver dystrophy, the doctor prescribed a lipotropic drug - a donor of methyl groups. This is likely:

Глюкоза Glucose

Білірубін Bilirubin

S-Аденозилметіонін S-Adenosylmethionine

Валін Valin

Холестерин Cholesterol

100 / 200
Хворому на гострий інфаркт міокарда у комплексній терапії було призначено гепарин. Через деякий час після введення даного препарату з’явилась гематурія. Який антагоніст гепарину необхідно ввести хворому для усунення даного ускладнення? A patient with an acute myocardial infarction was prescribed heparin in complex therapy. Some time after the administration of this drug, hematuria appeared. What heparin antagonist should be administered to the patient to eliminate this complication?

Амінокапронова кислота Aminocaproic acid

Вікасол Vikasol

Неодикумарин Neodicoumarin

Фібриноген Fibrinogen

Протаміну сульфат Protamine sulfate

101 / 200
У хворого виявили злоякісну анемію. Терапія внутрішньо-м’язовим введення вітаміну Б12 давала нетривалий нестійкий ефект поліпшення складу крові. Пацієнт - завзятий рибаль і часто вживає самостійно виловлену і недостатньо термічно оброблену рибу. Який діагноз можна припустити? The patient was diagnosed with pernicious anemia. Therapy with intramuscular administration of vitamin B12 gave a short-term, unstable effect of improving blood composition. The patient is an avid fisherman and often consumes self-caught and undercooked fish. What diagnosis can be assumed?

Анкілостомоз Ankylostomosis

Трихоцефальоз Trichocephalus

Парагонімоз Paragonimosis

Ентеробіоз Enterobiosis

Дифілоботріоз Diphyllobotriosis

102 / 200
У синтезі пуринових нуклеотидів беруть участь деякі амінокислоти, похідні вітамінів, фосфорні ефіри рибози. Коферментна форма якого вітаміну є переносником одновуглецевих фрагментів в цьому синтезі? In the synthesis of purine nucleotides, some amino acids, vitamin derivatives, phosphoric esters of ribose are involved. The coenzyme form of which vitamin is the carrier of one-carbon fragments in this synthesis?

Піридоксин Pyridoxine

Пантотенова кислота Pantothenic acid

Рибофлавін Riboflavin

Фолієва кислота Folic acid

Нікотинова кислота Nicotinic acid

103 / 200
У тварини зруйнували отолітові вестибулорецептори. Які з наведених рефлексів зникнуть внаслідок цього у тварини? The animal's otolith vestibuloreceptors were destroyed. Which of the following reflexes will disappear in the animal as a result?

Міотатичні Myotatic

Статокінетичні при рухах з кутовим прискоренням Statokinetic during movements with angular acceleration

Випрямлення тулуба Torso straightening

Первинні орієнтувальні Primary indicative

Статокінетичні при рухах з лінійним прискоренням Statokinetic when moving with linear acceleration

104 / 200
Лікар записав в історії хвороби, що у хворого дихання поверхневе (знижена глибина дихання). Це означає, що зменшеним є такий показник зовнішнього дихання: The doctor recorded in the medical history that the patient's breathing is shallow (reduced depth of breathing). This means that the external breathing rate is reduced:

Життєва ємність легень Vital lung capacity

Хвилинний об’єм дихання Minute respiratory volume

Функціональна залишкова ємність Functional residual capacity

Дихальний об’єм Respiratory volume

Ємність вдиху Inhalation capacity

105 / 200
До лікаря звернувся студент з проханням призначити препарат для лікування алергічного риніту, який виник у нього під час цвітіння липи. Який засіб можна застосувати? A student turned to the doctor with a request to prescribe a drug for the treatment of allergic rhinitis, which occurred during the flowering of the linden tree. What drug can be used?

Амброксол Ambroxol

Норадреналіну гідротартрат Noradrenaline hydrotartrate

Анаприлін Anaprilin

Лозартан Losartan

Лоратадин Loratadine

106 / 200
Хворий похилого віку страждає на хронічний закреп, в основі якого лежить гіпотонія товстої кишки. Який препарат слід призначити хворому? An elderly patient suffers from chronic constipation, the basis of which is hypotonia of the colon. What drug should be prescribed to the patient?

Касторова олія Castor oil

Натрію сульфат Sodium sulfate

Бісакодил Bisacodyl

Новокаїнамід Novocaineamide

Атропіну сульфат Atropine sulfate

107 / 200
При лабораторному дослідженні дитини виявлено підвищений вміст у крові та сечі лейцину, валіну, ізолейцину та їх кетопохідних. Сеча має характерний запах кленового сиропу. Недостатність якого ферменту характерно для цього захворювання? During the child's laboratory examination, an elevated content of leucine, valine, isoleucine and their keto derivatives was found in the blood and urine. The urine has a characteristic smell of maple syrup. The deficiency of which enzyme is characteristic of this disease?

Глюкозо-6-фосфатаза Glucose-6-phosphatase

Фосфофруктомутаза Phosphofructomutase

Фосфофруктокіназа Phosphofructokinase

Дегідрогеназа розгалужених амінокислот Dehydrogenase of branched-chain amino acids

Амінотрансфераза Aminotransferase

108 / 200
При обстеженні 2-х місячної дитини педіатр звернула увагу, що плач дитини нагадує котячий крик. Діагностовані мікроцефалія і вада серця. За допомогою цитогенетичного метода з’ясований каріотип дитини: 46, XX, 5р. Дане захворювання є наслідком такого процесу: During the examination of a 2-month-old child, the pediatrician noticed that the child's cry resembled a cat's cry. Microcephaly and a heart defect were diagnosed. The child's karyotype was determined using the cytogenetic method : 46, XX, 5yr. This disease is a consequence of the following process:

Інверсія Inversion

Делеція Deletion

Дуплікація Duplication

Плейотропія Pleiotropy

Транслокація Translocation

109 / 200
Під час аутопсії тіла хворого, котрий помер від тяжкого ексикозу, на ґрунті профузної діареї, виявлені такі зміни: слизова оболонка прямої та сигмоподібної кишок на всьому протязі вкрита сіро-білими плівчастими нашаруваннями, що міцно з’єднані з підлеглими тканинами, між плівками розташовані множинні великі й поверхневі виразки, вкриті згортками крові. Мікроскопічно діагностовано фібринозно-виразковий коліт. Яке захворювання проявляється такими змінами? During the autopsy of the body of a patient who died of severe exicosis, on the basis of profuse diarrhea, the following changes were found: the mucous membrane of the rectum and sigmoid intestine is covered with gray white membranous layers that are firmly connected to the underlying tissues, between the films there are multiple large and superficial ulcers covered with blood clots. Fibrinous-ulcerative colitis was diagnosed microscopically. What disease is manifested by such changes?

Колі-інфекція Coli infection

Дизентерія Dysentery

Ієрсініоз Yersiniosis

Сальмонельози Salmonellosis

Стафілококова інфекція Staphylococcal infection

110 / 200
У лікарню до кінця робочого дня доставлений працівник 'гарячого' цеху, який скаржиться на головний біль, запаморочення, нудоту, загальну слабкість. Свідомість збережена, шкірні покриви гіперемовані, сухі, гарячі на дотик. ЧСС- 130/хв. Дихання часте, поверхневе. Яке порушення процесів регуляції тепла найімовірніше виникло у людини у даній ситуації? By the end of the working day, an employee of a 'hot' workshop was taken to the hospital, complaining of headache, dizziness, nausea, general weakness. Consciousness is preserved, the skin is hyperemic, dry, hot to the touch. Heart rate - 130/min. Breathing is frequent, shallow. What violation of heat regulation processes most likely occurred in a person in this situation?

Зниження тепловіддачі Reduction of heat transfer

Посилення тепловіддачі і теплопродукції Increasing heat transfer and heat production

Посилення тепловіддачі і зниження теплопродукції Increasing heat transfer and decreasing heat production

Посилення теплопродукції без змін тепловіддачі Increased heat production without changes in heat output

Зниження теплопродукції без зміни тепловіддачі Decreasing heat production without changing heat output

111 / 200
В експерименті подразнюють гілочки симпатичного нерва, які іннервують серце. Це призвело до збільшення сили серцевих скорочень, тому що через мембрану типових кардіоміоцитів збільшився: In the experiment, the branches of the sympathetic nerve, which innervate the heart, are irritated. This led to an increase in the force of heart contractions, because through the membrane of typical cardiomyocytes increased:

Вихід іонів кальцію Output of calcium ions

Вхід іонів кальцію Input of calcium ions

Вхід іонів калію Input of potassium ions

Вхід іонів кальцію та калію Input of calcium and potassium ions

Вихід іонів калію Output of potassium ions

112 / 200
У пацієнта 60-ти років виявлено збільшення порогу сприймання звуків високої частоти. Зміна функцій яких структур слухового аналізатора зумовлює виникнення цього порушення? In a 60-year-old patient, an increase in the threshold for the perception of high-frequency sounds was detected. A change in the functions of which auditory analyzer structures causes this disturbance?

Барабанної перетинки Eardrum

Євстахієвої труби Eustachian tube

М’язів середнього вуха Middle ear muscles

Органу Корті ближче до овального віконця The organ of Corti is closer to the oval window

Органу Корті ближче до гелікотреми The organ of Corti is closer to the helicotrema

113 / 200
У 3-річної дитини тривале підвищення температури, збільшені лімфовузли, у крові - значне підвищення кількості лімфоцитів. Методом ІФА виявлено антиген віруса Епштейна-Бара. Який діагноз можна поставити на основі вказаного? A 3-year-old child has a long-term fever, enlarged lymph nodes, a significant increase in the number of lymphocytes in the blood. Epstein-Barr virus antigen was detected by ELISA. What diagnosis can be made on the basis of the specified?

Інфекційний мононуклеоз Infectious mononucleosis

Лімфома Беркета Burkett's lymphoma

Іерпетична аденопатія Herpetic adenopathy

Іенералізована інфекція, викликана herpes-zoster Generalized infection caused by herpes-zoster

Цитомегаловірусна інфекція Cytomegalovirus infection

114 / 200
У дитини виявлена схильність до ожиріння, яка є результатом діатезу. Назвіть вид діатезу, при якому частіше може розвинутись ожиріння: The child has a tendency to obesity, which is the result of a diathesis. Name the type of diathesis in which obesity can develop more often:

Лімфатико-гіпопластичний Lymphatic-hypoplastic

Нервово-артритичний Nervous-arthritic

Ексудативно-катаральний Exudative-catarrhal

Астенічний Asthenic

- -

115 / 200
У юнака 18-ти років діагностовано хворобу Марфана. При дослідженні встановлено порушення розвитку сполучної тканини, будови кришталика ока, аномалії серцево-судинної системи, арахнодактилію. Прикладом якого генетичного явища є ця хвороба? An 18-year-old young man was diagnosed with Marfan's disease. During the examination, a violation of the development of connective tissue, the structure of the lens of the eye, abnormalities of the cardiovascular system, and arachnodactyly were found. An example of which genetic phenomenon is this disease?

Плейотропія Pleiotropy

Неповне домінування Incomplete dominance

Кодомінування Codominance

Множинний алелізм Multiple allelism

Комплементарність Complementarity

116 / 200
Ураження хворого одноразовою дозою іонізуючого випромінювання спричинило розвиток кістково-мозкової форми променевої хвороби. Які патологічні прояви з боку крові будуть характерними в період удаваного благополуччя? Injury of the patient with a single dose of ionizing radiation caused the development of the bone-cerebral form of radiation sickness. What pathological manifestations on the part of the blood will be characteristic during the period of feigned well-being?

Тромбоцитопенія, лейкоцитоз Thrombocytopenia, leukocytosis

Перерозподільчий лейкоцитоз, лім-фоцитоз Redistributive leukocytosis, lymphocytosis

Тромбоцитопенія, анемія Thrombocytopenia, anemia

Анемія, лейкопенія Anemia, leukopenia

Наростаюча лімфопенія, лейкопенія Increasing lymphopenia, leukopenia

117 / 200
У хворого на черевний тиф при проведенні серологічного дослідження (реакція Відаля) виявлено О- і Н-аглютитиніни у титрі 1:800 і 1:200 відповідно. Це свідчить про: O- and H-agglutitinins were detected in the titer of 1:800 and 1:200, respectively, in a patient with typhoid fever during a serological examination (Vidal's reaction). This indicates about:

Раніше перенесене захворювання Previous illness

Проведене щеплення Vaccination carried out

Неможливість підтвердити діагноз Unable to confirm diagnosis

Період реконвалесценції Period of convalescence

Початок захворювання Start of disease

118 / 200
У дитячому колективі проведено планову вакцинацію проти кору. Яким методом можна перевірити ефективність проведеної вакцинації? A scheduled vaccination against measles was carried out in the children's team. What method can be used to check the effectiveness of the vaccination?

Вірусоскопічний Virusoscopic

Біологічний Biological

Алергопроба Allergy test

Серологічний Serological

Вірусологічний Virus

119 / 200
У клінічній практиці застосовують для лікування туберкульозу препарат ізоніазид - антивітамін, який здатний проникати у туберкульозну палочку. Туберкулостатичний ефект обумовлений порушенням процесів реплікації, окисно-відновних реакцій завдяки утворенню несправжнього коферменту з: In clinical practice, the drug isoniazid is used for the treatment of tuberculosis - an antivitamin that is able to penetrate into the tubercle bacillus. The tuberculostatic effect is due to the violation of replication processes, redox reactions due to the formation of a false coenzyme with:

ФМН FMN

ФАД FAD

НАД OVER

ТДФ TDF

КоО KoO

120 / 200
При огляді хворої лікар-гінеколог відмітив симптоми запалення статевих шляхів, у мазку взятому із піхви, виявлено грушоподібні найпростіші з шипом, з передньої частини відходять джгутики, наявна ундулююча мембрана. Яке захворювання підозрює лікар у хворої? During the examination of the patient, the gynecologist noticed symptoms of inflammation of the genital tract, in a smear taken from the vagina, pear-shaped protozoa with a thorn were found, flagella come from the front part, an undulating membrane is present What disease does the doctor suspect in the patient?

Токсоплазмоз Toxoplasmosis

Урогенітальний трихомоноз Urogenital trichomoniasis

Лямбліоз Giardiasis

Балантидіоз Balantidiosis

Кишковий трихомоноз Intestinal trichomoniasis

121 / 200
У чоловіка 32-х років, хворого на пневмонію, спостерігається закупорка харкотинням дихальних шляхів. В організмі хворого при цьому буде розвиватися така зміна кислотно-лужної рівноваги: A 32-year-old man with pneumonia has a sputum blockage of the respiratory tract. At the same time, the patient's body will develop the following change in acid-alkaline balance:

Метаболічний алкалоз Metabolic alkalosis

Респіраторний ацидоз Respiratory acidosis

Метаболічний ацидоз Metabolic acidosis

Респіраторний алкалоз Respiratory alkalosis

Змін не буде There will be no changes

122 / 200
При аналiзi ЕКГ виявлено випадіння деяких серцевих циклів PQRST. Наявні зубці та комплекси не змінені. Назвать вид аритмії: During the ECG analysis, the loss of some PQRST cardiac cycles was detected. The existing teeth and complexes are not changed. Name the type of arrhythmia:

Синоатріальна блокада Sinoatrial block

Передсердна екстрасистола Atrial extrasystole

Атріовентрикулярна блокада Atrioventricular block

Внутрішньопередсердна блокада Intra-atrial blockade

Миготлива аритмія Atrial fibrillation

123 / 200
У хворої встановлено порушення виділення тиреотропного гормону гіпофіза. Зі зниженням функцій якої частки гіпофіза це пов’язано? The patient has been diagnosed with a violation of the release of thyroid-stimulating hormone from the pituitary gland. This is due to a decrease in the functions of which part of the pituitary gland?

Infundibulum Infundibulum

- -

Lobus posterior Lobus posterior

Lobus anterior Lobus anterior

Pars intermedia Pars intermedia

124 / 200
Стресовий стан і больове відчуття у пацієнта перед візитом до стоматолога супроводжуються анурією (відсутністю сечовиділення). Це явище зумовлене збільшенням: The patient's stress and pain before visiting the dentist are accompanied by anuria (absence of urination). This phenomenon is caused by an increase in:

Секреції вазопресину та адреналіну Secretions of vasopressin and epinephrine

Активності парасимпатичної нервової системи Activities of the parasympathetic nervous system

Активності антиноціцептивної системи Activities of the antinociceptive system

Секреції адреналіну та зменшенням вазопресину Secretions of epinephrine and reduction of vasopressin

Секреції вазопресину та зменшенням адреналіну Secretion of vasopressin and reduction of adrenaline

125 / 200
Введення знеболюючого пацієнту перед екстракцією зуба призвело до розвитку анафілактичного шоку, який супроводжувався розвитком олігурії. Який патогенетичний механізм зумовив зменшення діурезу в даній клінічній ситуації? The administration of an anesthetic to a patient before tooth extraction led to the development of anaphylactic shock, which was accompanied by the development of oliguria. What pathogenetic mechanism caused the decrease in diuresis in this clinical situation?

Зменшення кількості функціонуючих нефронів Decreasing the number of functioning nephrons

Зниження гідростатичного тиску в капілярах клубочків Reduction of hydrostatic pressure in glomerular capillaries

Збільшення онкотичного тиску крові Increased blood oncotic pressure

Підвищення гідростатичного тиску в капсулі Шумлянського-Боумена Increase in hydrostatic pressure in the Shumlyansky-Bowman capsule

Пошкодження клубочкового фільтру Damage of the glomerular filter

126 / 200
Хвора на ревматоїдний артрит після трьохтижневого лікування преднізолоном почала скаржитись на перебої в роботі серця. З чим пов’язаний розвиток даного небажаного ефекту препарату? After a three-week treatment with prednisolone, a patient with rheumatoid arthritis began to complain of interruptions in the work of the heart. What is the development of this undesirable effect of the drug?

Гіперурікемія Hyperuricemia

Гіперглікемія Hyperglycemia

Гіпоглікемія Hypoglycemia

Гіперкаліємія Hyperkalemia

Гіпокаліємія Hypokalemia

127 / 200
У хворого 45-ти років на тлі трансмурального інфаркту міокарда розвинулася гостра лівошлуночкова недостатність. Який лікарський засіб доцільно застосувати у даній ситуації для покращення помпової функції серця? A 45-year-old patient developed acute left ventricular failure against the background of a transmural myocardial infarction. What medicine should be used in this situation to improve the pumping function of the heart?

Ефедрин Ephedrine

Добутамін Dobutamine

Промедол Promedol

Еуфілін Euphilin

Ізадрин Izadrin

128 / 200
Для профілактики атеросклерозу, ішемічної хвороби серця, порушень мозкового кровообігу рекомендується споживання жирів із високим вмістом поліненасичених жирних кислот. Однією з таких жирних кислот є: For the prevention of atherosclerosis, coronary heart disease, disorders of cerebral circulation, it is recommended to consume fats with a high content of polyunsaturated fatty acids. One of these fatty acids is:

Олеїнова Oleinova

Лауринова Laurinova

Лінолева Linoleum

Стеаринова Stearinova

Пальмітоолеїнова Palmitoolein

129 / 200
У хворого з нагноєнням рани при бактеріологічному дослідженні ранового вмісту виявлено грамнегативну паличку, яка на МПА утворює напівпрозорі слизові колонії синьо-зеленого кольору з перламутровим відтінком. Культура має специфічний запах фіалок або жасмину. Який вид збудника виділений з рани хворого? In a patient with suppuration of a wound, a gram-negative bacillus was found during bacteriological examination of the wound contents, which on MPA forms translucent mucous colonies of blue-green color with a mother-of-pearl shade. The culture has a specific smell violets or jasmine. What type of pathogen was isolated from the patient's wound?

P vulgaris P vulgaris

S. aureus S. aureus

S. pyogenes S. pyogenes

P aeruginosa P aeruginosa

S. faecalis S. faecalis

130 / 200
Після обстеження пацієнта в клініці нервових хвороб встановлена відсутність звуження зіниці при дії світла. З ураженням яких структур головного мозку це пов’язано? After examining the patient in the clinic of nervous diseases, it was established that there is no constriction of the pupil when exposed to light. What brain structures is this affected?

Червоні ядра середнього мозку Red nuclei of the midbrain

Ядра гіпоталамуса Nuclei of the hypothalamus

Вегетативні ядра 3 пари черепно-мозкових нервів Vegetative nuclei of 3 pairs of cranial nerves

Ретикулярш ядра довгастого мозку Reticularis of the nucleus of the medulla oblongata

Ретикулярні ядра середнього мозку Reticular nuclei of the midbrain

131 / 200
У хворого під час комп’ютерної томографії грудної клітки діагностовано пухлину заднього нижнього середостіння. Яка з перерахованих структур стиснута пухлиною? The patient was diagnosed with a tumor of the posterior lower mediastinum during a chest CT scan. Which of the listed structures is compressed by the tumor?

Aorta thoracica Aorta thoracica

Vena cava superior Vena cava superior

Arcus aortae Arcus aortae

N. phrenicus N. phrenicus

Trachea Trachea

132 / 200
Під час операції холецистектомії у хірурга виникла необхідність визначити топографію загальної жовчної протоки. Злиттям яких проток утворюється дана анатомічна структура? During the cholecystectomy operation, the surgeon needed to determine the topography of the common bile duct. The fusion of which ducts forms this anatomical structure?

Лівої печінкової і міхурової проток Left hepatic and cystic duct

Загальної печінкової і лівої печінкової проток Common hepatic and left hepatic duct

Загальної печінкової і правої печінкової проток Common hepatic and right hepatic duct

Правої та лівої печінкових проток Right and left hepatic ducts

Загальної печінкової і міхурової проток Common hepatic and cystic duct

133 / 200
Дитина 6-ти років під час гри порізала ногу осколком скла і була направлена у поліклініку для введення протиправцевої сироватки. З метою попередження розвитку анафілактичного шоку лікувальну сироватку вводили за методом Безредка. Який механізм лежить в основі подібного способу гіпосенсибілізації організму? A 6-year-old child cut his leg with a shard of glass while playing and was sent to the polyclinic for administration of anti-tetanus serum. In order to prevent the development of anaphylactic shock, the therapeutic serum was administered using the method Bezredka. What mechanism underlies this method of hyposensitization of the body?

Зв’язування фіксованих на тучних клітинах IgE Mat cell-fixed IgE binding

Стимуляція синтезу антиген-специфічних IgG Stimulation of synthesis of antigen-specific IgG

Зв’язування рецепторів до IgE на тучних клітинах IgE receptor binding on mast cells

Стимуляція імунологічної толерантності до антигену Stimulation of immunological tolerance to antigen

Блокування синтезу медіаторів у тучних клітинах Blocking of synthesis of mediators in mast cells

134 / 200
Хворому лікар призначив протикашльовий препарат центральної дії, який є алкалоїдом мачку жовтого. Діє на кашльовий центр вибірково, не пригнічуючи дихання, не затримує виділення харкотиння. Не викликає обстипації і лікарської залежності. Можна призначати дітям. Визначте препарат: The doctor prescribed an antitussive drug of central action to the patient, which is an alkaloid of yellow maca. It acts on the cough center selectively, without suppressing breathing, and does not delay the release of sputum. It does not cause constipation and drug addiction. Can be prescribed to children. Identify the drug:

Глауцину гідрохлорид Glaucine hydrochloride

Кодеїну фосфат Codeine Phosphate

Бромгексин Bromhexine

Лібексин Libexin

Окселадин Oxeladine

135 / 200
Хлопчику 5-ти років був встановлений діагноз - міастенія. Оберіть препарат з групи антихолінестеразних засобів, який покращує нервово-м’язову передачу: A 5-year-old boy was diagnosed with myasthenia gravis. Choose a drug from the group of anticholinesterase drugs that improves neuromuscular transmission:

Ацеклідин Aceclidine

Армін Armin

Алоксим Aloxim

Іалантаміну гідробромід Yalanthamine hydrobromide

Прозерин Proserin

136 / 200
Хворому 63-х років з атонією сечового міхура лікар призначив препарат, дозу якого хворий самостійно збільшив. З’явились підвищене потовиділення, салівація, діарея, м’язові спазми. Препарат якої групи був призначений? The doctor prescribed a drug to a 63-year-old patient with atony of the bladder, the dose of which the patient increased on his own. Increased sweating, salivation, diarrhea, muscle spasms appeared What group of drugs was prescribed?

Токолітики Tocolytics

Холіноміметики Cholinomimetics

Адреноблокатори Adrenoblockers

Гангліоблокатори Ganglioblockers

Реактиватори холінестерази Cholinesterase reactivators

137 / 200
Під час розтину тіла жінки 28-ми років, яка померла від геморагічного шоку, виявлено: гемоперитонеум, права маткова труба збільшена у розмірах, багряна, з наскрізним дефектом стінки, її дилятований просвіт виповнений темно-червоними згортками крові. Гістологічно у слизовій оболонці труби та серед м’язових клітин визначаються пласти великих світлих децидуальних клітин, у м’язовій оболонці та серед згортків крові у просвіті труби - ворсинки хоріона. Діагностуйте патологію вагітності: During the autopsy of the body of a 28-year-old woman who died of hemorrhagic shock, it was found: hemoperitoneum, the right fallopian tube was enlarged, purple, with a through wall defect , its dilated lumen is filled with dark red blood clots. Histologically, in the mucous membrane of the tube and among the muscle cells, layers of large light decidual cells are determined, in the muscle membrane and among the blood clots in the lumen of the tube - chorionic villi. Diagnose the pathology of pregnancy:

Черевна вагітність Abdominal pregnancy

Порушена трубна вагітність Disrupted tubal pregnancy

Інтерлігаментарна вагітність Interligamentous pregnancy

Порушена маткова вагітність Disturbed uterine pregnancy

Деструюючий міхурцевий занесок Destructive Bubble Drift

138 / 200
У біоптаті нирки 45-річного чоловіка, що має хронічну хворобу нирок, виявлено: склероз, лімфо-плазмоцитарна інфільтрація стінок мисок та чашок, дистрофія та атрофія канальців. Збережені канальці розширені, розтягнені колоїдоподібними масами, епітелій сплющений (’’щитоподібна” нирка). Який діагноз найбільш імовірний? A kidney biopsy of a 45-year-old man with chronic kidney disease revealed: sclerosis, lympho-plasmacytic infiltration of the walls of bowls and cups, dystrophy and atrophy of tubules. Saved the tubules are dilated, stretched by colloid-like masses, the epithelium is flattened ('thyroid' kidney). What is the most likely diagnosis?

Гострий пієлонефрит Acute pyelonephritis

Хронічний пієлонефрит Chronic pyelonephritis

Тубуло-інтерстиційний нефрит Tubulo-interstitial nephritis

Нефросклероз Nephrosclerosis

Гломерулонефрит Glomerulonephritis

139 / 200
Чоловік 40 років хворіє на гіперацидний гастрит з нічними голодними болями. Призначте хворому лікарський засіб - блокатор гістамінових H2-рецепторів III покоління, який знизить виділення хлористоводневої кислоти (особливо вночі) та збільшить утворення захисного слизу: A 40-year-old man suffers from hyperacid gastritis with nocturnal hunger pangs. Prescribe the patient a medication - a histamine H2-receptor blocker of the III generation, which will reduce the release of hydrochloric acid (especially at night ) and will increase the formation of protective mucus:

Атропіну сульфат Atropine sulfate

Метацин Metacin

Фамотидин Famotidine

Пірензепін Pirenzepine

Платифіліну гідротартрат Platiphylline hydrotartrate

140 / 200
Під час футбольного матчу між вболівальниками різних команд виникла сутичка. На фоні негативних емоцій в одного учасника сутички були розширені зіниці й підвищене серцебиття. Активація якої системи регуляції функцій організму забезпечує такі вегетативні зміни при негативних емоціях? During a football match, a fight broke out between fans of different teams. Against the background of negative emotions, one participant in the fight had dilated pupils and an increased heart rate. Activation of which system of regulation of body functions provides such autonomic changes during negative emotions?

Парасимпатична нервова Parasympathetic nerve

Метасимпатична нервова Metasympathetic nerve

Симпато-адреналова Sympathoadrenal

Соматична нервова Somatic nerve

Гіпоталамо-гіпофізарно-тиреоїдна Hypothalamus-pituitary-thyroid

141 / 200
У хворого 32-х років після оперативного втручання розвинувся гнійний процес. Із гною рани виділено культуру S. aureus. Який з перерахованих тестів найдоцільніше використати для диференціації S. aureus від S. epidermidis? A 32-year-old patient developed a purulent process after surgery. A culture of S. aureus was isolated from the pus of the wound. Which of the listed tests is most appropriate to use for differentiation of S. aureus from S. epidermidis?

Оксидазний тест Oxidase test

Колір колонії Colony Color

Ферментація арабінози Arabinose fermentation

Гемоліз на кров’яному агарі Hemolysis on blood agar

Плазмокоагулазна активність Plasmocoagulase activity

142 / 200
Чоловікові 58-ми років зроблено операцію з приводу раку простати. Через 3 місяці йому проведено курс променевої та хіміотерапії. До комплексу лікарських препаратів входив 5-фтордезоксиуридин - інгібітор тимідилатсинтази. Синтез якої речовини блокується цим препаратом? A 58-year-old man was operated on for prostate cancer. After 3 months, he underwent a course of radiation and chemotherapy. The complex of drugs included 5-fluorodeoxyuridine - a thymidylate synthase inhibitor . The synthesis of which substance is blocked by this drug?

і-РНК i-RNA

ДНК DNA

- -

т-РНК t-RNA

р-РНК p-RNA

143 / 200
В підводному човні під час занурення порушилася система подачі кисню. У підводників збільшилися частота дихання і серцевих скорочень. Який вид гіпоксії розвинувся у підводників? The oxygen supply system was broken in the submarine during the dive. The breathing and heart rate of the submariners increased. What type of hypoxia developed in the submariners?

Серцево-судинна Cardiovascular

Кров’яна Bloody

Дихальна Respiratory

Гіпоксична Hypoxic

Тканинна Fabric

144 / 200
Синтез і-РНК проходить на матриці ДНК з урахуванням принципу комплементарності. Якщо триплети у ДНК наступні - АТГ-ЦГТ, то відповідні кодони і-РНК будуть: The synthesis of i-RNA takes place on the DNA matrix taking into account the principle of complementarity. If the triplets in DNA are as follows - ATG-CHT, then the corresponding codons of i-RNA will be:

УАЦ-ГЦА UAC-HCA

УАГ-ЦГУ UAG-CSU

АУГ-ЦГУ AUG-CSU

ТАГ-УГУ TAG-UGU

АТГ-ЦГТ ATG-CHT

145 / 200
У процесі фібринолізу кров’яний тромб розсмоктується. Розщеплення нерозчинного фібрину відбувається шляхом його гідролізу під дією протеолітичного ферменту плазміну, який наявний у крові в неактивній формі плазміногену. Активується плазміноген шляхом обмеженого протеолізу за участю фермента: In the process of fibrinolysis, the blood clot dissolves. Insoluble fibrin is broken down by its hydrolysis under the action of the proteolytic enzyme plasmin, which is present in the blood in the inactive form of plasminogen. Plasminogen is activated by limited proteolysis with the participation of an enzyme:

Трипсин Trypsin

Пепсин Pepsin

Хімотрипсин Chymotrypsin

Ентерокіназа Enterokinase

Урокіназа Urokinase

146 / 200
У біоптаті щитоподібної залози виявлено атрофію паренхіматозних елементів, дифузну інфільтрацію тканини залози лімфоцитами та плазматичними клітинами з утворенням в ній лімфоїдних фолікулів. Для якого захворювання є характерними наведені ознаки? Atrophy of parenchymal elements, diffuse infiltration of the gland tissue by lymphocytes and plasma cells with the formation of lymphoid follicles were found in the biopsy of the thyroid gland. For which disease are these symptoms characteristic?

Ендемічний зоб Endemic goiter

Аденома щитоподібної залози Thyroid adenoma

Тиреоїдит Ріделя Riedel's thyroiditis

Тиреоїдит Хасімото Hashimoto's thyroiditis

Хвороба Базедова Bazedov's disease

147 / 200
У хворого з жовтяницею встановлено: підвищення у плазмі крові вмісту загального білірубіну за рахунок непрямого (вільного), в калі і сечі - високий вміст стеркобіліну, рівень прямого (зв’язаного) білірубіну у плазмі крові в межах норми. Який вид жовтяниці має місце у хворого? In a patient with jaundice, it was established: an increase in the content of total bilirubin in the blood plasma due to indirect (free), in feces and urine - a high content of stercobilin, the level of direct (with bilirubin in the blood plasma is within the normal range. What type of jaundice does the patient have?

Хвороба Жильбера Gilbert's disease

Гемолітична Hemolytic

- -

Механічна Mechanical

Паренхіматозна Parenchymatous

148 / 200
При гістологічному дослідженні біоптатів, взятих з потовщених країв виразки шлунка, виявлені невеликі гніздові скупчення різко атипових гіперхромних невеликих епітеліальних клітин, які розташовані серед дуже розвиненої строми. Визначте пухлину: On histological examination of biopsies taken from the thickened edges of a gastric ulcer, small nested clusters of sharply atypical hyperchromic small epithelial cells were found, which are located among a highly developed stroma. Identify the tumor:

Медулярний рак Medullary cancer

Аденома Adenoma

Недиференційована саркома Undifferentiated sarcoma

Скіррозний недиференційований рак Scarcerous undifferentiated cancer

Аденокарцинома Adenocarcinoma

149 / 200
Жінка 69-ти років довго хворіла на атеросклероз. Поступила в хірургічне відділення з симптомами гострого живота. При лапаротомії виявлені: тромбоз мезентеріальної артерії, петлі тонкої кишки набряклі, багряно-чорного кольору, на їх серозній оболонці фібринозні нашарування. Який патологічний процес розвинувся у кишці хворої? A 69-year-old woman suffered from atherosclerosis for a long time. She was admitted to the surgical department with acute abdominal symptoms. At laparotomy, the following were found: thrombosis of the mesenteric artery, the loops of the small intestine were swollen, purple - black in color, fibrinous layering on their serous membrane. What pathological process developed in the patient's intestine?

Немічний інфаркт Nemic heart attack

Коагуляційний некроз Coagulation necrosis

Суха гангрена Dry gangrene

Волога гангрена Wet gangrene

Секвестр Sequestration

150 / 200
Під час ректороманоскопії хворого зі скаргами на діарею виявлено, що слизова оболонка прямої і сигмоподібної кишок різко гіперемована, набрякла, вкрита великою кількістю слизу, а у деяких ділянках вкрита плівчастими накладаннями зеленуватого кольору. Про яке захворювання можна думати? During the rectoromanoscopy of a patient with complaints of diarrhea, it was found that the mucous membrane of the rectum and sigmoid intestine is sharply hyperemic, swollen, covered with a large amount of mucus, and in some areas covered with membranous greenish overlays. What disease can you think of?

Амебіаз Amebiasis

Дизентерія Dysentery

Холера Cholera

Сальмонельоз Salmonellosis

Черевний тиф Typhoid

151 / 200
При копрологічному дослідженні у працівників кав’ярні лікарями санітарно- епідеміологічної станції були виявлені округлі цисти, характерною ознакою яких є наявність чотирьох ядер. Імовірніше за все у цих працівників безсимптомно паразитує: During the coprological examination of coffee shop workers, the doctors of the sanitary and epidemiological station found rounded cysts, the characteristic feature of which is the presence of four nuclei. Most likely, these workers are asymptomatic parasitizes:

Кишкова трихомонада Intestinal trichomonas

Амеба кишкова Intestinal amoeba

Дизентерійна амеба Dysenteric amoeba

Лямблія Lamblia

Балантидій Balantidius

152 / 200
У хворого з варикозним розширенням вен під час огляду нижніх кінцівок відзначається: ціаноз, пастозність, зниження температури шкіри, поодинокі петехії. Який розлад гемодинаміки має місце у хворого? In a patient with varicose veins, during the examination of the lower extremities, the following is noted: cyanosis, pastiness, a decrease in skin temperature, isolated petechiae. What hemodynamic disorder does the patient have?

Тромбоемболія Thromboembolism

Обтураційна ішемія Obstructive ischemia

Компресійна ішемія Compression ischemia

Артеріальна гіперемія Arterial hyperemia

Венозна гіперемія Venous hyperemia

153 / 200
При гістологічному дослідженні органів і тканин померлої від ниркової недостатності молодої жінки, у якої прижиттєво виявлявся високий титр антинуклеарних антитіл, виявлені поширені фібриноїдні зміни в стінках судин. Відмічається ядерна патологія з вакуолізацією ядер, каріорексисом, утворенням гематоксилінових тілець. Який найбільш імовірний діагноз? During a histological examination of the organs and tissues of a young woman who died of kidney failure, in whom a high titer of antinuclear antibodies was detected during life, widespread fibrinoid changes in the vessel walls were detected. Nuclear pathology is noted with vacuolization of nuclei, karyorrhexis, formation of hematoxylin bodies. What is the most likely diagnosis?

Атеросклероз Atherosclerosis

Системний червоний вовчак Systemic lupus erythematosus

Гіпертонічна хвороба Hypertensive disease

Вузликовий періартеріїт Nodular periarteritis

Облітеруючий ендартеріїт Endarteritis obliterans

154 / 200
Мікоплазми є своєрідною групою мікроорганізмів, що відносяться до родини Mycoplasmataceae і що мають властивості як бактерій, так і вірусів. Назвіть одну особливість мікоплазм, яка відрізняє їх від бактерій і вірусів: Mycoplasmas are a peculiar group of microorganisms that belong to the family Mycoplasmataceae and have properties of both bacteria and viruses. Name one feature of mycoplasmas that distinguishes them from bacteria and viruses:

Відсутність клітинної стінки No cell wall

Спосіб розмноження Reproduction method

Відсутність клітинної будови No cellular structure

Висока ферментативна активність High enzymatic activity

Внутрішньоклітинний паразитизм Intracellular parasitism

155 / 200
Після довготривалого вживання антибіотиків у хворого на слизовій ротової порожнини появилися округлі білі плями, на язику білий наліт. Який мікроорганізм імовірно спричинив дані симптоми? After long-term use of antibiotics, the patient developed round white spots on the mucous membrane of the oral cavity, a white plaque on the tongue. What microorganism probably caused these symptoms?

Кишкова паличка Escherichia coli

Гриби роду Candida Candida fungi

Лактобацили Lactobacilli

Стрептокок Streptococcus

Ентерокок Enterococcus

156 / 200
Фенілкетонурія - це захворювання, яке зумовлено рецесивним геном, що локалізується в аутосомі. Батьки є гетерозиготами за цим геном. Вони вже мають двох хворих синів і одну здорову дочку. Яка імовірність, що четверта дитина, яку вони очікують народиться теж хворою? Phenylketonuria is a disease caused by an autosomal recessive gene. The parents are heterozygous for this gene. They already have two sick sons and one healthy daughter. What is the probability that the fourth child they expect will also be sick?

0% 0%

75% 75%

50% 50%

25% 25%

100% 100%

157 / 200
При диспансерному обстеженні у хворого знайдено цукор в сечі. Який найбільш імовірний механізм виявлених змін, якщо вміст цукру в крові нормальний? During the dispensary examination, sugar was found in the patient's urine. What is the most probable mechanism of the detected changes, if the blood sugar content is normal?

!нсулінорезистентність рецепторів клітин !insulin resistance of cell receptors

Недостатня продукція інсуліну підшлунковою залозою Insufficient production of insulin by the pancreas

Порушення фільтрації глюкози в клубочковому відділі нефрона Disturbance of glucose filtration in the glomerular part of the nephron

Порушення реабсорбції глюкози в канальцях нефрона Disturbance of glucose reabsorption in nephron tubules

Гіперпродукція глюкокортикоїдів наднирниками Hyperproduction of glucocorticoids by the adrenal glands

158 / 200
В пробірку, що містить розчин NaCl 0,9%, додана крапля крові. Що відбудеться з еритроцитами? A drop of blood was added to a test tube containing a 0.9% NaCl solution. What will happen to the erythrocytes?

Біологічний гемоліз Biological hemolysis

Осмотичний гемоліз Osmotic hemolysis

Зморшкування Wrinkle

Залишаться без змін Remain unchanged

Набухання Swelling

159 / 200
У хворого 40-ка років ознаки гірської хвороби: запаморочення, задишка, тахікардія, рН крові - 7,50, pCO2 - 30 мм рт.ст., зсув буферних основ +4 ммоль/л. Яке порушення кислотно-основного стану має місце? A 40-year-old patient has signs of mountain sickness: dizziness, shortness of breath, tachycardia, blood pH - 7.50, pCO2 - 30 mm Hg, shift buffer bases +4 mmol/l. What violation of the acid-base state is taking place?

Негазовий ацидоз Nongaseous acidosis

Газовий алкалоз Gas alkalosis

Негазовий алкалоз Nongaseous alkalosis

Газовий ацидоз Gas acidosis

Видільний ацидоз Excretory acidosis

160 / 200
У хворого 15-ти років концентрація глюкози натще - 4,8 ммоль/л, через годину після цукрового навантаження - 9,0 ммоль/л, через 2 години - 7,0 ммоль/л, через 3 години - 4,8 ммоль/л. Ці показники характерні для такого захворювання: A 15-year-old patient has a fasting glucose concentration of 4.8 mmol/l, an hour after a sugar load - 9.0 mmol/l, after 2 hours - 7.0 mmol/l, after 3 hours - 4.8 mmol/l. These indicators are typical for such a disease:

Цукровий діабет I типу Type I diabetes

Прихований цукровий діабет Hidden diabetes

Цукровий діабет II типу Type II diabetes

Хвороба !ценко-Кутінга. Tsenko-Kuting disease.

- -

161 / 200
Лікар призначив хворому з гострою серцевою недостатністю не-глікозидний кардіотонічний засіб, який безпосередньо стимулює в1-адренорецептори міокарда, що збільшує кровообіг, діурез. Застосовується лише внутрішньовенно крапельно внаслідок швидкої інактивації в організмі. Який препарат призначив лікар? The doctor prescribed a non-glycoside cardiotonic agent to a patient with acute heart failure, which directly stimulates β1-adrenoceptors of the myocardium, which increases blood circulation, diuresis. It is used only intravenously as a result of rapid inactivation in the body. What drug did the doctor prescribe?

Добутамін Dobutamine

Дигоксин Digoxin

Анаприлін Anaprilin

Адреналін Adrenaline

Корглікон Corglycon

162 / 200
Чоловік 40-ка років перебував у пульмонологічному відділенні з приводу рецидивуючої правосторонньої пневмонії. Помер від легенево-серцевої недостатності. На розтині в правій легені визначається ділянка круглої форми 3х4 см. Вона являє собою порожнину з нерівними шорсткими краями, заповнену каламутною вершкоподібною жовто-зеленою рідиною. Мікроскопічно: стінка порожнини утворена тканиною легені з дифузною інфільтрацією лейкоцитами. Визначте патологічний процес у легені: A 40-year-old man was in the pulmonology department due to recurrent right-sided pneumonia. He died of pulmonary heart failure. An autopsy revealed a round 3x4 cm area in the right lung . It is a cavity with uneven rough edges, filled with a cloudy creamy yellow-green liquid. Microscopically: the wall of the cavity is formed by lung tissue with diffuse infiltration of leukocytes. Define the pathological process in the lung:

Гангрена Gangrene

Емпієма Empyema

Хронічний абсцес Chronic abscess

Гострий абсцес Acute abscess

!нфаркт !heart attack

163 / 200
У вагітної жінки 24-х років після тривалого блювання було зареєстровано зниження об’єму циркулюючої крові. Про яку зміну загальної кількості крові може йти мова? A 24-year-old pregnant woman had a decrease in the volume of circulating blood after prolonged vomiting. What kind of change in the total amount of blood can we be talking about?

Олігоцитемічна гіперволемія Oligocythemic hypervolemia

Олігоцитемічна гіповолемія Oligocythemic hypovolemia

Поліцитемічна гіперволемія Polycythemic hypervolemia

Проста гіповолемія Simple hypovolemia

Поліцитемічна гіповолемія Polycythemic hypovolemia

164 / 200
Пацієнту, який знаходився в клініці з приводу пневмонії, ускладненої плевритом, у складі комплексної терапії вводили преднізолон. Протизапальна дія цього синтетичного глюкокортикоїда пов’язана з блокуванням вивільнення арахідонової кислоти шляхом гальмування: A patient who was in the clinic for pneumonia complicated by pleurisy was administered prednisolone as part of complex therapy. The anti-inflammatory effect of this synthetic glucocorticoid is associated with blocking the release of arachidonic acid by braking:

Циклооксигенази Cyclooxygenases

Пероксидази Peroxidases

Фосфоліпази С Phospholipase C

Ліпоксигенази Lipoxygenases

Фосфоліпази A2 Phospholipase A2

165 / 200
У чоловіка 64-х років, який тривалий час курив і вживав міцні спиртні напої, на боковій поверхні язика виявили подібний на виразку утвір із білої, помірно щільної тканини розмірами 5х3 см. При гістологічному дослідженні біоптату виявили, що утвір побудований із клітини, які формують солідні структури і тяжі, що нагадують за будовою багатошаровий плоский епітелій, в якому клітини з вираженим поліморфізмом, з великими атиповими ядрами з патологічними мітозами. Діагностуйте виявлене у чоловіка захворювання: In a 64-year-old man who smoked and drank strong alcoholic drinks for a long time, an ulcer-like formation of white, moderately dense tissue with the size of 5x3 cm. During the histological examination of the biopsy, it was found that the formation is made of cells that form solid structures and rods, resembling in structure a multilayered flat epithelium, in which cells with pronounced polymorphism, with large atypical nuclei with pathological mitoses. Diagnose the disease detected in a man :

Плоскоклітинний зроговілий рак Squamous keratinized cancer

Рак на місці Cancer in place

Плоскоклітинний незроговілий рак Squamous nonkeratinous cancer

Еритроплакія Erythroplakia

Лейкоплакія Leukoplakia

166 / 200
Обстежуваний знаходиться у фазі повільнохвильового глибокого сну. Про це свідчить реєстрація на ЕЕГ таких хвиль: The subject is in the phase of slow-wave deep sleep. This is evidenced by the registration of the following waves on the EEG:

Бета-хвилі Beta Waves

Тета -хвилі Theta Waves

Дельта-хвилі Delta Waves

Альфа-веретена Alpha Spindle

Альфа-хвилі Alpha Waves

167 / 200
Лікар-стоматолог для лікування гінгівіту призначив пацієнту препарат з протипротозойною та антибактеріальною діями, який може викликати відразу до алкоголю. Вкажіть препарат, який призначив лікар: To treat gingivitis, the dentist prescribed the patient a drug with antiprotozoal and antibacterial effects, which can cause an aversion to alcohol. Specify the drug prescribed by the doctor:

Цефтріаксон Ceftriaxone

Лінкоміцину гідрохлорид Lincomycin hydrochloride

Левоміцетин Levomycetin

Тетрациклін Tetracycline

Метронідазол Metronidazole

168 / 200
Спеціальний режим харчування призвів до зменшення іонів Ca2+ в крові. До збільшення секреції якого гормону це призведе? A special diet led to a decrease in Ca2+ ions in the blood. Which hormone will this lead to an increase in secretion?

Тирокальцитонін Tyrocalcitonin

Тироксин Thyroxine

Соматотропін Somatotropin

Паратгормон Parathyroid hormone

Вазопресин Vasopressin

169 / 200
У пацієнта після переохолодження на губах з’явились герпетичні висипання. Для лікування призначений крем ацикловіру, терапевтичний ефект якого пояснюється здатністю: The patient developed herpetic rashes on his lips after hypothermia. Acyclovir cream is prescribed for treatment, the therapeutic effect of which is explained by the ability to:

Гальмувати активність протеази Inhibit protease activity

Гальмувати активність ДНК-полімерази Inhibit DNA polymerase activity

Уповільнювати реплікацію РНК- і ДНК-геномних вірусів Slow down the replication of RNA and DNA genomic viruses

Інгібувати нейрамінідазу Inhibit neuraminidase

Порушувати функцію зворотної транскриптази Disrupt reverse transcriptase function

170 / 200
На плановий прийом до педіатра батьки привели дитину віком 13 місяців. Під час повного огляду лікар перевірив розвиток II сигнальної системи дитини. Назвіть період, коли у людини вперше з’являються ознаки розвитку II сигнальної системи: Parents brought a 13-month-old child to a pediatrician for a routine appointment. During a complete examination, the doctor checked the development of the child's II signal system. Name the period when a person first had there are signs of the development of the II signal system:

1,5-2 роки 1.5-2 years

2-2,5 роки 2-2.5 years

6-12 місяців 6-12 months

3-5 років 3-5 years

2,5-3 роки 2.5-3 years

171 / 200
У сироватці крові новонародженого виявлено антитіла до вірусу кору. Про наявність якого імунітету це може свідчити? Antibodies to the measles virus were detected in the blood serum of a newborn. What kind of immunity does this indicate?

Природний пасивний Natural Passive

Природний активний Natural Active

Штучний пасивний Artificial Passive

Штучний активний Artificial Active

Спадковий, видовий Hereditary, species

172 / 200
У хворого спостерігається пухлина тканин орбіти позаду очного яблука. Зазначено порушення акомодації та звуження зіниці ока. Яке анатомічне утворення ушкоджено? The patient has a tumor of the orbital tissues behind the eyeball. A violation of accommodation and narrowing of the pupil of the eye is noted. What anatomical formation is damaged?

Ganglion ciliare Ganglion ciliare

N. lacrimalis N. lacrimalis

N. nasociliaris N. nasociliaris

N. opticus N. opticus

N. trochlearis N. trochlearis

173 / 200
У хворого виявлено зміну функції привушної слинної залози. Який з вузлів вегетативної нервової системи віддає післявузлові симпатичні волокна для неї? A change in the function of the parotid salivary gland was detected in the patient. Which of the nodes of the autonomic nervous system sends postnodal sympathetic fibers to it?

Ganglion submandibulare Ganglion submandibulare

Ganglion oticum Ganglion oticum

Ganglion cervicothoracicum Ganglion cervicothoracicum

Ganglion cervicale superius Ganglion cervicale superius

Ganglion pterygopalatinum Ganglion pterygopalatinum

174 / 200
Жінка 31-го року хворіє на В!Л- інфекцію на стадії СНІД. На шкірі нижніх кінцівок, слизової оболонки піднебіння з’явились рудувато-червоні плями, яскраво-червоні вузлики різних розмірів. Один з вузликів взято на гістологічне дослідження. Виявлено багато хаотично розташованих тонкостінних судин, вистелених ендотелієм, пучки веретеноподібних клітин з наявністю ге-мосидерину. Яка пухлина розвинулась у хворої? A 31-year-old woman suffers from HIV infection at the AIDS stage. Reddish-red spots appeared on the skin of the lower extremities, the mucous membrane of the palate, bright -red nodules of various sizes. One of the nodules was taken for histological examination. Many thin-walled vessels lined with endothelium, bundles of spindle-shaped cells with the presence of hemosiderin were found. What kind of tumor developed in the patient?

Лімфома Беркіта Burkitt's lymphoma

Лімфангіома Lymphangioma

Гемангіома Hemangioma

Фібросаркома Fibrosarcoma

Саркома Капоші Kaposi's sarcoma

175 / 200
У альпініста, що піднявся на висоту 5200 м, розвинувся газовий алкалоз. Що є причиною його розвитку? A mountaineer who climbed to a height of 5200 m developed gas alkalosis. What is the cause of its development?

Гіповентиляція легенів Hypoventilation of the lungs

Зниження температури навколишнього середовища Decreasing ambient temperature

Гіпоксемія Hypoxemia

Гіпервентиляція легенів Hyperventilation of the lungs

Гіпероксемія Hyperoxemia

176 / 200
У хворого хронічна нежить. Набряк слизової оболонки носової порожнини призводить до порушення функції рецепторів нюхового нерва, які розташовані в нюховій ділянці носової порожнини. Через яке утворення волокна цього нерва потрапляють в передню черепну ямку? The patient has a chronic runny nose. Swelling of the mucous membrane of the nasal cavity leads to impaired function of the receptors of the olfactory nerve, which are located in the olfactory area of the nasal cavity. Through which formation do the fibers of this nerve enter in the front cranial fossa?

Foramen incisivum Foramen incisivum

Foramen sphenopalatinum Foramen sphenopalatinum

Lamina cribrosa os ethmoidale Lamina cribrosa os ethmoidale

Foramen ethmoidale posterior Foramen ethmoidale posterior

Foramen ethmoidale anterior Foramen ethmoidale anterior

177 / 200
На гістологічному препараті в складі видовженої структури, обмеженої плазмолемою, по периферії розташовані численні ядра, а в цитоплазмі наявна поперечна посмугованість. Яка це структура? On the histological preparation, as part of an elongated structure, limited by the plasmolemma, numerous nuclei are located on the periphery, and transverse striations are present in the cytoplasm. What is this structure?

Гладкий міоцит Smooth myocyte

Кардіоміоцит Cardiomyocyte

Синцитіотрофобласт Syncytiotrophoblast

Міосимпласт Myosymplast

Колагенове волокно Collagen fiber

178 / 200
У спортсмена легкоатлета (бігуна на довгі дистанції) під час змагань розвинулась гостра серцева недостатність. В результаті чого виникла ця патологія? A track and field athlete (long-distance runner) developed acute heart failure during the competition. What caused this pathology?

Перевантаження серця об’ємом Heart volume overload

Порушення вінцевого кровообігу Violation of coronary blood circulation

Прямого пошкодження міокарда Direct myocardial damage

Патологія перикарда Pericard pathology

Перевантаження серця опором Heart overload with resistance

179 / 200
У пацієнта встановлено гіповітаміноз фолієвої кислоти, що може призвести до порушення синтезу: The patient is diagnosed with hypovitaminosis of folic acid, which can lead to impaired synthesis:

Цитрату та кетонових тіл Citrate and ketone bodies

Пуринових нуклеотидів та холестерину Purine nucleotides and cholesterol

Гема та креатину Heme and creatine

Пуринових та тимідилових нуклеотидів Purine and thymidyl nucleotides

Тимідилових нуклеотидів та жирних кислот Thymidyl nucleotides and fatty acids

180 / 200
Спадкова гіперліпопротеїнемія І типу обумовлена недостатністю ліпопротеїнліпази. Підвищення рівня яких транспортних форм ліпідів в плазмі навіть натщесерце є характерним? Hereditary hyperlipoproteinemia of type I is caused by lipoprotein lipase deficiency. An increase in the level of which transport forms of lipids in the plasma even on an empty stomach is characteristic?

Модифіковані ліпопротеїни Modified lipoproteins

Ліпопротеїни високої густини High-density lipoprotein

Ліпопротеїни дуже низької густини Very low density lipoprotein

Ліпопротеїни низької густини Low-density lipoprotein

Хіломікрони Chylomicrons

181 / 200
У людини виявлена пухлина одного з відділів головного мозку, внаслідок чого в неї порушена здатність підтримувати нормальну температуру тіла. Яка структура головного мозку пошкоджена? A person has a tumor in one of the brain departments, as a result of which his ability to maintain normal body temperature is impaired. What brain structure is damaged?

Таламус Thalamus

Стріатум Striatum

Гіпоталамус Hypothalamus

Мозочок Cerebellum

Чорна субстанція Black substance

182 / 200
До лікарні звернувся чоловік 50-ти років з розладами пам’яті, болісними відчуттями по ходу нервових стовбурів, зниженням інтелектуальних функцій, порушеннями з боку серцево-судинної системи і явищами диспепсії. В анамнезі хронічний алкоголізм. Дефіцит якого вітаміну може викликати ці симптоми? A 50-year-old man came to the hospital with memory disorders, painful sensations along the course of the nerve trunks, a decrease in intellectual functions, disorders of the cardiovascular system and phenomena of dyspepsia. There is a history of chronic alcoholism. Which vitamin deficiency can cause these symptoms?

Ніацин Niacin

Тіамін Thiamine

Ретинол Retinol

Кальциферол Calciferol

Рибофлавін Riboflavin

183 / 200
Пацієнт звернувся зі скаргами на гострий біль у правому підребер’ї. При огляді лікар звернув увагу на пожовтіння склер хворого. Лабораторно: підвищена активність АлАТ та негативна реакція на стеркобілін в калі. Для якого захворювання характерні такі симптоми? The patient complained of acute pain in the right hypochondrium. During the examination, the doctor noted the yellowing of the patient's sclera. Laboratory: increased activity of ALT and a negative reaction to stercobilin in feces. What disease is characterized by such symptoms?

Гепатит Hepatitis

Хронічний гастрит Chronic gastritis

Гемолітична жовтяниця Hemolytic Jaundice

Хронічний гастродуоденіт Chronic gastroduodenitis

Хронічний коліт Chronic Colitis

184 / 200
У хворого після тривалого психоемоційного напруження спостерігається підвищення артеріального тиску, що супроводжується серцебиттям, кардіалгіями, головним болем, запамороченням. Домінуючим у формуванні артеріальної гіпертензії у даному випадку є збільшення: After prolonged psycho-emotional stress, the patient has an increase in blood pressure, which is accompanied by palpitations, cardiac pain, headache, dizziness. The dominant factor in the formation of arterial hypertension in this case is an increase in:

Тонусу венул Tonus venules

Серцевого викиду Cardiac output

Тонусу артеріол Arteriolar tone

Об’єму циркулюючої крові Circulating blood volume

Частоти серцевих скорочень Heart rate

185 / 200
Незалежно від расової чи етнічної належності у людини розвивається комплекс морфофункціональних, біохімічних, імунологічних ознак, які обумовлюють кращу біологічну пристосованість людини до відповідного фізичного середовища. Який тип біологічної реакції представлений у людини? Regardless of race or ethnicity, a person develops a complex of morphofunctional, biochemical, and immunological features that cause better biological adaptation of a person to the corresponding physical environment. What type of biological reaction is presented in a person?

Гірський тип Mountain type

Адаптивний тип Adaptive type

Тропічний тип Tropical type

Тип зони помірного клімату Temperate climate zone type

Арктичний тип Arctic type

186 / 200
Для знеболювання використовують новокаїн, під дією якого нервове волокно втрачає здатність проводити збудження. Який мембранно-іонний механізм дії цього препарату? For pain relief, novocaine is used, under the influence of which the nerve fiber loses its ability to conduct excitation. What is the membrane-ion mechanism of action of this drug?

Блокування натрій-протонного насоса Na-proton pump blocking

Блокування кальцієвих іоноселектив-них каналів Blocking of calcium ion-selective channels

Блокування калієвих іоноселективних каналів Blocking of potassium ion-selective channels

Блокування калій-натрієвого насоса Potassium sodium pump blocking

Блокування натрієвих іоноселективних каналів Blocking of sodium ion-selective channels

187 / 200
У хворого 41-го року відзначається гіпонатріємія, гіперкаліємія, дегідратація, зниження артеріального тиску, м’язова слабкість, брадикардія, аритмія. З порушенням функцій яких гормонів це пов’язано? A 41-year-old patient has hyponatremia, hyperkalemia, dehydration, decreased blood pressure, muscle weakness, bradycardia, and arrhythmia. What hormones are this associated with 'linked?

Гормони підшлункової залози Pancreatic hormones

Кортикостероїди Corticosteroids

Статеві гормони Sex hormones

Гормони мозкової речовини наднирників Adrenal medulla hormones

Тиреоїдні Thyroid

188 / 200
При розтині хворої 28-ми років, що померла від уремії, виявлені збільшені строкаті нирки з осередками крововиливів. Патогістологічно в судинних клубочках виявлені гематоксилінові тільця, капілярні мембрани клубочків у вигляді дротяних петель, гіалінові тромби та осередки фібриноїдного некрозу. За патогенезом гіперчутливість якого типу лежить в основі описаної хвороби? At the autopsy of a 28-year-old patient who died of uremia, enlarged variegated kidneys with foci of hemorrhages were found. Pathohistologically, hematoxylin bodies were found in vascular glomeruli, capillary membranes of glomeruli in in the form of wire loops, hyaline thrombi and foci of fibrinoid necrosis. What type of hypersensitivity is the basis of the described disease in terms of pathogenesis?

Гіперчутливість IV типу (клітинна цитотоксичність) Type IV hypersensitivity (cellular cytotoxicity)

Гіперчутливість II типу (антитілозалежна) Type II hypersensitivity (antibody dependent)

Гіперчутливість I типу (анафілактична) Type I hypersensitivity (anaphylactic)

Гіперчутливість III типу (імунокомплексна) Type III hypersensitivity (immunocomplex)

Гіперчутливість V типу (гранулематоз) V type hypersensitivity (granulomatosis)

189 / 200
Хвора 37-ми років померла під час нападу експіраторної задухи, що був спричинений контактом з екзогенним алергеном (пилок амброзії). При гістологічному дослідженні в просвіті бронхів спостерігаються скупчення слизу, в стінці бронхів багато тучних клітин (лаброцитів), більшість з яких у стані де-грануляції, багато еозинофілів. До патогенезуякого типу реакцій гіперчутливості можна віднести описані зміни? A 37-year-old patient died during an attack of expiratory suffocation, which was caused by contact with an exogenous allergen (ambrosia pollen). During histological examination, accumulations of mucus are observed in the lumen of the bronchi , there are many mast cells (labrocytes) in the wall of the bronchi, most of which are in a state of degranulation, many eosinophils. The described changes can be attributed to the pathogenesis of what type of hypersensitivity reactions?

V типу (гранулематоз) Type V (granulomatosis)

II типу (антитілозалежна) Type II (antibody dependent)

IV типу (клітинна цитотоксичність) Type IV (cellular cytotoxicity)

I типу (анафілактична) Type I (anaphylactic)

III типу (імунокомплексна) Type III (immunocomplex)

190 / 200
Оперуючи на наднирниковій залозі, хірург зупиняє кровотечу з артерії, що відходить до наднирникової залози від черевної аорти. Назвіть цю артерію: When operating on the adrenal gland, the surgeon stops the bleeding from the artery leading to the adrenal gland from the abdominal aorta. Name this artery:

A. suprarenalis media A. suprarenalis media

A. suprarenalis superior A. suprarenalis superior

A. phrenica inferior A. phrenica inferior

A. uprarenalis inferior A. uprarenalis inferior

A. renalis A. renalis

191 / 200
Хворому на паратиф А інфекціоніст на 3-му тижні захворювання призначив повторне бактеріологічне дослідження. Який матеріал слід взяти для виділення збудника? On the 3rd week of the disease, the infectious disease specialist prescribed a repeat bacteriological examination for a patient with paratyphoid A. What material should be taken to isolate the pathogen?

Харкотиння Snoring

Випорожнення Peel

Ліквор Liquor

Кров Blood

Блювотні маси Vomiting masses

192 / 200
Пацієнтка тривалий час приймала снодійний засіб нітразепам. Після відміни препарату у неї розвинулись безсоння, зниження апетиту, агресивність. Як називається такий стан? The patient has been taking the hypnotic nitrazepam for a long time. After stopping the drug, she developed insomnia, decreased appetite, and aggressiveness. What is this condition called?

Ейфорія Euphoria

Сенсибілізація Sensitization

Тахіфілаксія Tachyphylaxis

Абстиненція Abstinence

Кумуляція Cumulative

193 / 200
Хворому на шизофренію призначено аміназин для купіювання психічного стану. Вкажіть механізм дії препарату: A patient with schizophrenia is prescribed aminazine to buy a mental state. Specify the mechanism of action of the drug:

Стимуляція опіоїдних рецепторів Stimulation of opioid receptors

Стимуляція серотонінових рецепторів Serotonin receptor stimulation

Блокада ГАМК-рецепторів GABA receptor blockade

Блокада Д2-дофамінових рецепторів Blockade of D2-dopamine receptors

- -

194 / 200
Хворому на пневмонію призначений антибіотик із групи макролідів. Вкажіть цей препарат: A patient with pneumonia is prescribed an antibiotic from the macrolide group. Specify this drug:

Азитроміцин Azithromycin

Тетрациклін Tetracycline

Гентаміцин Gentamicin

Ампіцилін Ampicillin

Стрептоміцин Streptomycin

195 / 200
В клініку доставлено чоловіка з травмою спини. Під час обстеження виявлено перелом хребців грудного відділу. Під час об’єктивного огляду нейрохірургом виявлено: нижче рівня перелому з правого боку відсутня глибока чутливість, з лівого боку – порушена температурна та тактильна чутливість. Яке ураження з боку спинного мозку є у хворого? A man was brought to the clinic with a back injury. During the examination, a fracture of the thoracic vertebrae was detected. During an objective examination by a neurosurgeon, it was found: below the level of the fracture on the right side, there is no deep sensitivity, on the left side - impaired temperature and tactile sensitivity. What damage does the patient have on the side of the spinal cord?

Анестезія Anesthesia

Хвороба Паркінсона Parkinson's disease

Парастезія Paraesthesia

Синдром Броун Секара Brown Secard syndrome

Судомний синдром Convulsive syndrome

196 / 200
Через 8 днів після хірургічної операції у пацієнта розвинувся правець. Лікар запідозрив, що причиною став контамінований збудником правця шовний матеріал, який був доставлений в бактеріологічну лабораторію. Яке живильне середовище необхідно використовувати для первинного посіву шовного матеріалу? After 8 days after the surgical operation, the patient developed tetanus. The doctor suspected that the cause was suture material contaminated with the tetanus pathogen, which was delivered to the bacteriological laboratory. What a nutrient medium must be used for primary sowing of suture material?

Кітт-Тароцці Kitt-Tarozzi

Сабуро Saburo

Гіса His

Ендо Endo

ЖСА ЖСА

197 / 200
Після проведеної операції на шиї хворий втратив чутливість в її передній ділянці. Яка гілка шийного сплетення була пошкоджена під час операції? After an operation on the neck, the patient lost sensitivity in its front area. Which branch of the cervical plexus was damaged during the operation?

Малий потиличний нерв Small occipital nerve

Надключичні нерви Supracral nerves

Шийна петля Neck Loop

Поперечний нерв шиї Transverse neck nerve

Великий вушний нерв Great auricular nerve

198 / 200
У пацієнта перед кардіологічною операцією зареєстровано тиск у всіх відділах серця. Який тиск в лівому шлуночку під час діастоли? The pressure in all parts of the heart was recorded in the patient before cardiac surgery. What is the pressure in the left ventricle during diastole?

80 мм рт.ст. 80 mm Hg

100 мм рт.ст. 100 mm Hg

0 мм рт.ст. 0 mmHg

40 мм рт.ст. 40 mm Hg

120 мм рт.ст. 120 mm Hg

199 / 200
При обстеженні жінки виявили перелом однієї з кісток черепа, ускладнений кровотечею з поперечної пазухи твердої оболонки мозку. Назвіть кістку, на якій розташовується борозна цієї пазухи: During the woman's examination, a fracture of one of the bones of the skull was discovered, complicated by bleeding from the transverse sinus of the dura mater. Name the bone on which the groove of this sinus is located:

Скронева Temple

Тім’яна Timyana

Клиноподібна Wedge

Потилична Occipital

Лобова Lobova

200 / 200
Батьки - глухонімі, але глухота у дружини залежить від аутосомно-рецесивного гена, а у чоловіка виникла внаслідок тривалого прийому антибіотиків у дитинстві. Яка імовірність народження глухої дитини в родині, якщо батько гомозиготний за аллелю нормального слуху? Parents are deaf and mute, but the wife's deafness depends on an autosomal recessive gene, and the husband's was due to prolonged use of antibiotics in childhood. What is the probability of a deaf child being born in the family , if the father is homozygous for the normal hearing allele?

25% 25%

100% 100%

10% 10%

75% 75%

0% 0%